Contracts MBE Questions

अब Quizwiz के साथ अपने होमवर्क और परीक्षाओं को एस करें!

An auto manufacturer contracted with a supplier to provide speakers for 10,000 vehicles at a total price of $600,000. Prior to the date fixed for delivery of the speakers, the supplier, without justification, informed the manufacturer that it could not supply the speakers. The manufacturer immediately sought quotes from other suppliers. The manufacturer received a quote from a second supplier, who had previously provided the manufacturer with speakers in a timely and satisfactory manner, to sell 10,000 speakers for $800,000. A short time later, the manufacturer received an offer from a third supplier, with whom the manufacturer had not previously worked: to supply the 10,000 speakers for $600,000. The manufacturer, reasonably concerned that the third supplier would be unable to provide the speakers in a timely and satisfactory manner, entered into a contract with the second supplier. Subsequently, the first supplier told the manufacturer that it would be able to supply the speakers by the original delivery date and, although not requested by the manufacturer, provided the manufacturer with adequate evidence that the speakers would be timely delivered. The manufacturer told the first supplier not to deliver the goods and instead acquired them from the second supplier. The auto manufacturer sued the first supplier for $200,000. Should the auto manufacturer prevail? A. Yes, because the auto manufacturer is entitled to recover the cost of acquiring substitute speakers. B. Yes, because the first supplier had repudiated its contractual obligation. C. No, because the auto manufacturer was required to mitigate its damages by accepting the third supplier's lower offer. D. No, because the first supplier withdrew its repudiation prior to the time for performance.

Answer choice A is correct. A buyer can seek replacement goods and can recover the difference between the contract price and the cost of the replacement goods (i.e., cover). Here, the auto manufacturer paid $800,000 to acquire the speakers that the first supplier was contractually obligated to deliver for $600,000. Consequently, the manufacturer could recover $200,000 from the first supplier. Answer choice B is incorrect because a party who repudiates a contractual obligation prior to the time for its performance may nevertheless retract that repudiation and provide performance as called for in the contract. Here, although the first supplier had informed the manufacturer that the supplier would not deliver the speakers, the first supplier could have retracted that repudiation since the time for performance had not occurred. Answer choice D is incorrect because, while a party may retract an anticipatory repudiation, such retraction is prohibited where the other party has materially changed position as a consequence of the repudiation. Here, the manufacturer entered into a contract with the second supplier to acquire the speakers prior to the first supplier's retraction of its repudiation. Answer choice C is incorrect because, although a party is required to mitigate its damages, a party does not have to do so where doing so would involve undue risk. Here, obtaining the speakers from the third supplier would involve such risk.

A widow, struggling with a mortgage and crippling debt, was on the verge of bankruptcy. The widow recently inherited an undeveloped tract of land. Hoping to avoid losing her home, the widow advertised the land for sale online. A contractor responded with an offer to purchase the land. However, hoping to bargain with the inexperienced seller, the contractor initially offered to purchase the land for 15% of the fair market value of the land. The widow, desperate to avoid foreclosure on her home, immediately accepted the contractor's first offer. The widow and contractor recorded the agreement in a signed writing. Which of the following provides the best argument for the widow to cancel the contract? A. Unconscionability B. Unclean hands C. Lack of consideration D. Duress

Answer choice A is correct. A court may modify or refuse to enforce a contract or part of a contract because it is unconscionable. A contract is unconscionable when it is so unfair to one party that no reasonable person in the position of the parties would have agreed to it. Because of the unequal bargaining power in this transaction, the widow's desperation, and the incredibly low value the widow received for her property, the court may find that this contract was unconscionable at the time it was formed. Note that this situation is not necessarily unconscionable, but it is the widow's best argument to cancel the contract of the choices given. Answer choice B is incorrect because unclean hands is an equitable defense to raise against enforcement of a contract when the nonbreaching party is also guilty of some wrongdoing in the transaction at issue. Answer choice C is incorrect because there was consideration to support this contract, even if that consideration had a low economic value. Courts generally will not void a contract for merely inadequate consideration. Answer choice D is incorrect because duress is an improper threat that deprives a party of meaningful choice. Here, the contractor made no actual threat to force the widow to accept his offer. The widow's financial situation is insufficient to constitute a threat for the purposes of a duress defense.

A professor listed her car for sale on an online site. A buyer called and asked a series of questions, including the following, "I am not much of a music lover, but how does the stereo sound?" The professor replied that it sounded great. In reality, the stereo had short-circuited months earlier and blown out all the speakers in the process. Based on the conversation, the buyer entered into a written contract with the professor to purchase the car at a specified price. The contract provided for the buyer to pay for the car and take possession of it within three days. The next day, remorseful over her lie, the professor had the stereo and speakers restored to factory condition. On the day the buyer was supposed to take possession of the car, he learned from a student that the professor had lied about the condition of the stereo, and decided not to pay for the car. When the professor called the buyer later that day, the buyer told her he was not purchasing the car. The professor admitted her lie, and told the buyer that the stereo and speakers had been replaced. Nevertheless, the buyer refused to pay for the car.In a subsequent breach of contract suit, who will prevail? A. The professor, because she repaired the stereo so that it conformed to the buyer's expectations. B. The professor, because the condition of the stereo was not material to the buyer. C. The buyer, because the professor acted in bad faith in inducing the buyer to enter into the contract. D. The buyer, because he had the right to terminate the contract following the professor's misrepresentation.

Answer choice A is correct. A misrepresentation is an untrue assertion of fact. Fraudulent misrepresentation requires proof of the following: (i) the misrepresentation is made knowingly and with intent to mislead the other party (i.e., it is fraudulent), (ii) the misrepresentation induced assent to the contract, and (iii) the adversely affected party justifiably relied on the misrepresentation. A fraudulent misrepresentation need not be material, and may make the contract voidable at the adversely affected party's option. The contract is no longer voidable, however, if, following a misrepresentation but before the deceived party has avoided the contract, the facts are cured so as to be in accord with the facts that were previously misrepresented. Here, although the professor made a fraudulent misrepresentation, she cured the misrepresentation by having the stereo restored to factory condition before the buyer avoided the contract. Therefore, the contract is binding on the buyer, and his failure to pay constitutes a breach. Answer choice B is incorrect because a fraudulent misrepresentation need not be material for the contract to be voidable. Moreover, a misrepresentation is considered material if it is likely to induce a reasonable person to manifest assent or the seller knows that the misrepresentation is likely to induce the buyer himself to manifest his assent. In this case, the misrepresentation would likely be considered material. Answer choice C is incorrect because a misrepresentation can be cured at any point before the adversely affected party avoids the contract. Answer choice D is incorrect because the buyer's right to terminate the contract was subject to whether the professor had first cured the defects underlying her misrepresentation.

After a novice runner registered for an upcoming marathon, she went to a shoe store that specialized in shoes for runners. The runner informed a clerk at the store of her plans and of her need for appropriate shoes to train for the marathon. The clerk excused himself, went to the storeroom, and returned by mistake with shoes that were improper for long-distance running. The clerk, who was engaged in a conversation with another customer, handed the shoes to the runner but said nothing to her. The runner tried on the shoes and found that they fit. Impatient to leave, she took the shoes to the register and paid for them without talking with the clerk or any other store employee about the suitability of the shoes for her specific purpose. The cashier did notice that the shoes were on sale and told the runner that the shoes were sold with no warranties. The runner purchased the shoes anyway. The runner wore the shoes while training for the marathon. She suffered serious injury to her feet as a consequence. The runner has sued the store for breach of the warranty of fitness for a particular purpose. Is she likely to succeed? A. Yes, because the clerk knew that the runner intended to use the shoes for marathon training, and the runner relied on the clerk's knowledge. B. Yes, because the shoe store where the runner purchased the running shoes that were not suitable for marathon training was a merchant of running shoes. C. No, because the store employees did not make any express promise to the runner regarding the running shoes she purchased. D. No, because the cashier's comment constituted an effective disclaimer of any implied warranty.

Answer choice A is correct. A warranty that the goods are fit for a particular purpose is implied whenever the seller has reason to know (from any source, not just from the buyer) that the buyer has a particular use for the goods, and that the buyer is relying upon the seller's skill to select the goods. Here, the runner went not just to a store that sold shoes, but to one that specialized in selling running shoes. The store clerk was aware of the runner's need for shoes that were appropriate for running long distances, but instead mistakenly supplied the runner with shoes that did not meet that need. Consequently, the store has breached the warranty of fitness for a particular purpose. Answer choice B is incorrect. Although the store was a merchant for running shoes, any seller may breach the warranty of fitness for a particular purpose. Unlike the implied warranty of merchantability, the seller need not be a merchant. Answer choice C is incorrect because the warranty of fitness for a particular purpose is an implied warranty. There is no requirement that the seller make an express promise in order to violate this warranty. Answer choice D is incorrect. Unlike the implied warranty of merchantability, implied warranties of fitness for a particular purpose may be excluded by general language, but only if it is in writing and conspicuous. Here, the cashier's disclaimer was not in writing.

A customer entered a hardware store to purchase paint. The associate who helped the customer had just been hired the day before, and was not trained with regard to paint selection. The customer explained that she was painting her house and had applied a water-based primer, and that she needed to select an appropriate top coat. The associate showed the customer an oil-based paint, assuring the customer that the paint was appropriate for the job. The associate then told the customer that he was selling the paint "as is," and that he could not be responsible for any adverse reactions. The customer made the purchase and used the oil-based paint to paint her house. Upon drying, the paint immediately peeled away from the water-based primer, causing extensive damage to the exterior of the house.In a breach of warranty action against the hardware store, will the customer prevail? A. Yes, because the associate's promises created an express warranty. B. Yes, because the warranty of fitness for a particular purpose cannot be disclaimed. C. No, because the associate's statement that the paint was sold "as is" disclaimed any warranties. D. No, because the associate was not a merchant.

Answer choice A is correct. Any promise, affirmation, description, or sample that is part of the basis of the bargain is an express warranty, unless it is merely the seller's opinion or commendation of the value of the goods. In this case, the associate's assurances that the paint selected was appropriate for the job created an express warranty. That warranty was breached, and the customer would prevail. Answer choice B is incorrect. An implied warranty of fitness for a particular purpose can be disclaimed by conspicuous language in writing or by the use of language such as "as is," which calls the buyer's attention to the exclusion of warranties and makes plain that there is no implied warranty. Answer choice C is incorrect because disclaimer clauses that conflict with express warranties are ignored. Thus, any attempt to disclaim this express warranty with the language "as is" would be ignored. Answer choice D is incorrect because express warranties may be made by a merchant or a non-merchant. Moreover, the associate in this case was a merchant.

In anticipation of the holiday season, a toy store owner sent a signed, written order for 100 dolls at a cost of $100 per doll to a manufacturer. The manufacturer initially refused to accept the order due to the owner's bad credit, but shipped the doll to the owner when an investor, who had previously loaned the toy store owner $25,000 in order to keep the store afloat, called the manufacturer and guaranteed that if the owner failed to pay for the dolls, she would do so. The investor was worried that without this wildly popular doll, the store would not attract sufficient customers for the holidays, and the investor would lose her investment when the owner was forced to close the store. Due to discounts offered on the doll by larger competing retailers, the owner only sold ten dolls. The owner paid the manufacturer $1,000 for the dolls that were sold, but refused to pay the remaining $9,000. When the manufacturer contacted the investor for payment, the investor also refused to pay the manufacturer the remaining $9,000.If the manufacturer sues the investor to recover the $9,000, will it likely prevail? A. Yes, because the investor's purpose in agreeing to pay the manufacturer was to protect her investment in the toy store. B. Yes, because the benefit received by the investor was financial in nature. C. No, because the investor's promise was not in writing. D. No, because the investor did not receive the dolls.

Answer choice A is correct. Generally, a promise made to a person (i.e., the obligee) that the promisor (i.e., a surety) will be responsible for any debt or other obligation of a third party (i.e., the principal) resulting from the principal's failure to pay as agreed is subject to the Statute of Frauds, and the surety's promise must be in writing. However, if the main purpose of a surety in agreeing to pay the debt of the principal is the surety's own economic advantage, rather than the principal's benefit, then the contract does not fall within the Statute of Frauds, and an oral promise by the surety is enforceable. Here, because the investor entered into the suretyship promise because she wanted to protect her investment in the store, her oral promise is enforceable. Accordingly, answer choice C is incorrect. Answer choice B is incorrect because, although the investor did receive a financial benefit in promising to guarantee payment, consideration for a party's promise need not be financial in nature for that promise to be enforceable. Answer choice D is incorrect because a promise may be enforceable even though the primary beneficiary of the promise is a third party (here, the owner) rather than the promisor (here, the investor).

A painter entered into a valid contract with a mayor to paint his portrait for $5,000. The contract provided that, rather than paying the painter, the mayor was to pay the $5,000 to the painter's son. The contract also prohibited the assignment of any rights or duties arising under the contract without the permission of the other party. Subsequently, the son, upon learning of the contract from his mother, decided to donate this money to a charity and transferred his rights under the contract to the charity without securing the permission of the mayor. The painter painted the portrait, but the mayor did not pay anyone. The charity filed suit against the mayor for breach of contract. The charity has taken no action nor incurred any expenses related to the contract. Will the charity prevail in its action against the mayor? A. Yes, because the charity was an assignee of the son's contractual rights. B. Yes, because the charity is a donee beneficiary of the contract. C. No, because the charity has not detrimentally relied on the mayor's promise. D. No, because the contract prohibits assignments.

Answer choice A is correct. Generally, contract rights are assignable unless the assignment materially increases the duty or risk of the obligor or materially reduces the obligor's chance of obtaining performance. Here, because the right assigned is the right to receive payment and the painter has performed his obligation under the contract by painting the portrait, this right may be assigned and enforced by the charity through a breach of contract action. Answer choice B is incorrect because, although the charity does benefit from the contract by virtue of the son's gratuitous assignment, the charity is not a donee beneficiary of the contract because the painter and mayor did not enter into contract with the intent to benefit the charity. Answer choice C is incorrect because, as assignee of the son's contractual right to receive payment from the mayor, the charity can enforce those rights without the need to establish that it detrimentally relied on the mayor's promise. Answer choice D is incorrect because, although the assignment of contractual rights in violation of a prohibition against the assignment of rights does constitute a breach of contract, prohibition is not enforceable against the assignee to prevent the assignee from enjoying the assigned rights.

A company entered into a written agreement to design a home constructed of logs for a buyer. As part of the agreement, the company was to purchase and prepare logs for the home and then deliver them to land owned by the buyer. The buyer was responsible for providing the equipment for removing the logs from the company's tractor-trailers and constructing the home using the logs and plans provided by the company. The agreement called for the buyer to make a final payment to the company prior to delivery of the logs. The company prepared the design plans and the logs, but before the buyer made his final payment, the company demanded to know how the buyer planned on removing the logs from its tractor-trailers. The company made this demand in good faith based on safety concerns, but in the mistaken belief that it had the right to do so under the contract. When the buyer refused to comply, the company informed the buyer that it would not deliver the logs. As a consequence, the buyer refused to make his final payment under the contract. Which party has breached its contractual obligation? A. The company, because it anticipatorily repudiated its contractual duty to deliver the logs. B. The company, because it demanded that the buyer provide information that the buyer was not contractually obligated to provide. C. The buyer, because he refused to comply with the company's demand to disclose how the buyer intended to remove the logs from the company's tractor-trailers. D. The buyer, because he refused to make his final payment under the contract.

Answer choice A is correct. If a party to a contract clearly and unequivocally repudiates its contractual duty prior to its obligation to perform, the party has committed an anticipatory breach of the contract. A party's demand for performance for a term not contained in the contract, accompanied by an unequivocal statement that the demanding party will not perform a contractual duty unless the other party meets the additional term, constitutes an anticipatory breach of contract and excuses performance by the other party. Here, the company, mistakenly believing it had the right to do so, demanded that the buyer disclose how he planned on removing the logs from the company's tractor-trailers. When the buyer refused to comply, the company's unequivocal statement that it would not deliver the logs constituted an anticipatory breach of its contractual obligation. Answer choice B is incorrect. Although the company did not have a contractual right to demand the disclosure of this information from the buyer, this demand, by itself, did not constitute a breach of its contractual obligations. (Note: The fact that the company acted in good faith in doing so is not a defense.) Answer choice C is incorrect because the buyer was not under a contractual duty to disclose how the buyer intended to remove the logs from the company's tractor-trailers. Consequently, the buyer's refusal to make this disclosure did not constitute a breach of the contract. Answer choice D is incorrect. Although the buyer's contractual duty to make a final payment was a constructive condition to the company's duty to deliver the logs, the company unequivocally repudiated its contractual duty to deliver the logs and thereby excused the buyer's duty to make his final payment.

An adult daughter called a local restaurant to place a large delivery order. The restaurant generally requires a credit card for all delivery orders, but the daughter's father, who is a regular at the restaurant and happened to be there when the daughter placed the order, told the clerk that, in the event the daughter failed to pay for the food, he would do so. The restaurant delivered the order to the daughter, who, having decided to order something else instead, refused to accept or pay for the food. Can the restaurant collect from the father? A. No, because the father's promise was made orally. B. No, because a third party will not be held liable for the contract obligations of another. C. Yes, because the father promised to pay. D. Yes, because a parent is liable to pay for necessities provided to a child.

Answer choice A is correct. The Statute of Frauds applies to suretyship agreements (i.e., one person's promise to pay the debts or be responsible for an obligation of another). Here, the daughter ordered food and was obligated to pay for the food. However, the father also promised to pay for the food if the daughter did not. This promise created a suretyship agreement. Since this agreement was not in writing, the restaurant cannot enforce it. Note that some oral suretyship contracts can be enforced (indemnity contracts and contracts wherein the surety's main reason for paying the debt is the surety's own economic advantage), but those circumstances are not present here. Answer choice B is incorrect because third parties may be liable for the debts of another if they agree to be sureties. In such cases the surety agreement must generally be in writing. Answer choice C is incorrect because, although the father did promise to pay for the food if his daughter failed to do so, the Statute of Frauds applies to a suretyship agreement. Since the agreement was not in writing the father's promise is unenforceable. Answer choice D is incorrect because, although a parent is liable for necessities, such as food, provided by a third party to a minor child, a parent is not liable for necessities provided to an adult daughter or son.

A salon owner contacted a manufacturer by email about purchasing shampoo sinks. The manufacturer sent the salon owner the following email: "I will sell you four shampoo sinks at a discounted price of $300 apiece." The salon owner responded immediately, rejecting the offer. However, due to a transmission problem in the internet routing system, the message was not delivered to the manufacturer until the following day. In the meantime, the salon owner contacted several other sellers, all of whom made significantly higher offers. The salon owner then sent another email to the manufacturer, stating, "I accept your offer." This email was delivered immediately. The following day, the misrouted rejection email arrived in the manufacturer's inbox. Assume the parties are in a jurisdiction that applies the mailbox rule to electronic communications. Was a contract formed? A. Yes, because the salon owner accepted the manufacturer's offer. B. Yes, because the mailbox rule applies. C. No, because the salon owner rejected the offer prior to accepting the offer. D. No, because the manufacturer received the salon owner's rejection.

Answer choice A is correct. The mailbox rule states that a timely sent acceptance is effective when sent, not upon receipt. However, if a communication is sent rejecting the offer, and a later communication is sent accepting the contract, the mailbox rule does not apply, and the first one to be received by the offeror prevails. Here, the salon owner rejected the offer, then sent an acceptance. Since the acceptance was received by the offeror first, the acceptance prevails. Answer choice B is incorrect because, as noted with respect to answer choice A, the mailbox rule does not apply when an acceptance is sent after a rejection. Answer choice C is incorrect because, while a rejection's effect is to terminate an offer, to be effective, the rejection must be communicated to the offeror. When a rejection is sent before an acceptance, the mailbox rule does not apply and the first communication received by the offeror prevails. Answer choice D is incorrect because a contract was formed when the manufacturer received the acceptance before the rejection. The fact that the manufacturer received the rejection the following day may affect the damages to which the manufacturer is entitled if the salon owner refuses to honor the contract, but it does not control the issue of the formation of a contract.

A chicken farmer raised chickens to sell to people who wanted to eat fresh, organic, cage-free eggs at a lower cost than buying from a store. A nutritionist who was interested in raising chickens contacted the chicken farmer. The two entered into a written agreement that contained a clause stating that the agreement was the final and complete agreement between them. Prior to finalizing the contract, the farmer and the nutritionist had spoken on the phone, and they orally agreed that the farmer would not sell the chickens to the nutritionist unless she built a chicken coop for the chickens. After the contract was signed, the nutritionist attempted to have a chicken coop built, but due to the difficulty of building one in her small backyard, the coop would not be completed until after the chickens were delivered. On the day of delivery, the farmer refused to sell the chickens to the nutritionist when he saw that the chicken coop was not habitable. Is the farmer in breach of contract? A. No, because building the chicken coop was a condition precedent. B. No, because the agreement to build the chicken coop was oral, not written. C. Yes, because the nutritionist made a good-faith effort to build the coop. D. Yes, because the oral agreement occurred before the contract was finalized.

Answer choice A is correct. The parol evidence rule generally prevents a party to a written contract from presenting prior extrinsic evidence that contradicts the terms of the contract as written. However, parol evidence may be admitted to prove a condition precedent to the existence of the contract. Here, the farmer agreed to sell the chickens to the nutritionist on the condition that she build a chicken coop to house the chickens. Although the oral agreement regarding the chicken coop occurred prior to the finalization of the contract, it is admissible as a condition precedent. Thus, the farmer is not in breach of contract because the nutritionist did not satisfy the condition precedent to the existence of the contract for the chickens. For this reason, answer choice D is incorrect. Answer choice B is incorrect because a condition precedent does not have to be written; it can also be oral. Answer choice C is incorrect. Despite the nutritionist's good-faith effort to build the coop, she has not satisfied the condition precedent. Consequently, the farmer is not in breach of contract.

The owner of a failing café and a well-known restaurateur signed a written document indicating that the restaurateur would "pay for the transfer of all kitchen equipment within the kitchen at the café to the restaurant by December 31." After the cafe closed, the restaurateur went to the café and transferred all kitchen equipment to his own new restaurant site. The restaurateur researched the cost of the equipment at the time it was purchased and mailed a check to the café's owner for that amount. He had entered into many similar agreements with other businesses before, without any problems. The café owner immediately sent a letter to the restaurateur demanding the return of the kitchen equipment. The café owner argued that no contract was formed because the parties had not agreed to price and that the owner had anticipated a much higher sales price. Have the parties formed a valid and enforceable contract? A. Yes, because the court may supply the missing price term. B.Yes, because the restaurateur had entered into many similar agreements in the past. C. No, because a contract cannot be formed when there are missing terms. D. No, because the court had not supplied the missing price prior to performance.

Answer choice A is correct. Under the UCC a court can find that a contract for the sale of goods was formed if (1) the parties intended to contract, and (2) there is an objective standard to fill in any missing term. Here, the objective standard for the omitted price term (and the one presumably intended by the parties) was the fair market value of the restaurant equipment. Note, though that the terms typically used by the buyer are not an objective standard for the court to reference. Answer choice B is incorrect because the court cannot use the terms typically used by one party as a standard to apply to both parties. Answer choice C is incorrect because generally, the court may supply this term if there is an objective standard the court can reference. Answer choice D is incorrect because there is no such requirement.

A mother was in a canoe with her young son on a river. As the son leaned over the edge of the canoe to watch some ducks swim by, he fell into the river. The son could not swim and began to drown, so the mother jumped into the river to save him. However, she could not find him in the choppy water. A nearby fisherman saw the incident, jumped into the water, and saved the drowning boy by carrying him to the riverbank. In doing so, the fisherman was forced to drop the expensive fishing pole he was using to catch fish in the river, losing it forever. When the woman met the fisherman on the riverbank, she thanked him and promised to repay him for the lost fishing pole within the week. A week later, when the fisherman asked for the money, the mother changed her mind and told him she would not pay for his fishing pole. Assuming that none of the parties can be found negligent, under the common law, can the fisherman recover the cost of his fishing pole from the mother? A. No, because the mother's promise to pay for the fishing pole is not supported by consideration. B. No, because the son, and not the mother, received a material benefit from the fisherman. C. Yes, because the fisherman can recover under an implied-in-law contract. D. Yes, because the fisherman suffered a detriment when he dropped his fishing pole to save the son.

Answer choice A is correct. Under the common law, something given in the past is typically not good consideration because it could not have been bargained for, nor could it have been done in reliance upon a promise. Therefore, the fisherman's rescue of the son cannot be valid consideration for an enforceable contract between the mother and the fisherman. Answer choice B is incorrect. Most courts conclude that consideration exists if there is a detriment to the promisee, irrespective of the benefit to the promisor. A minority of courts look to either a detriment or a benefit, not requiring both. The Second Restatement asks only whether there was a bargained-for exchange. Regardless of what approach applies here, the mere fact that the mother did not receive the benefit under the contract would be insufficient to conclude that there was no valid consideration. Additionally, she did receive the benefit of not losing her son. Answer choice C is incorrect. When a plaintiff confers a benefit upon a defendant and the plaintiff has a reasonable expectation of compensation, allowing the defendant to retain the benefit without compensating the plaintiff would be unjust. In that case, the court can permit the plaintiff to recover the value of the benefit to prevent unjust enrichment. Here, even if the fisherman reasonably expected compensation for his rescue of the boy, the proper remedy would be the value of the benefit he conferred upon the mother (i.e., the value of the son's life). Therefore, even if the fisherman successfully brought a claim under an implied-in-law contract theory of recovery, the proper remedy would be the value of the benefit as restitution, not the costs he incurred in conferring the benefit. Answer choice D is incorrect because even though the fisherman suffered a detriment, it was not a bargained-for detriment. Therefore, his past act performed before the mother promised to pay for his fishing pole cannot be valid consideration to enforce that promise.

A man learned that his ex-wife might lose their former marital home, which she had been awarded outright as part of the divorce decree, due to her inability to make the monthly mortgage payment of $2,500. He sent her a letter promising to send her $2,500 in a week "for the house."Upon receipt of the letter, the ex-wife entered into a contract to purchase a big screen TV for $2,500. Upon learning of the contract for the TV, the man told his ex-wife he would not be sending her any money. She has sued him for $2,500. Will the ex-wife prevail? A. No, because repudiating the promise to give his ex-wife $2,500 did not result in injustice to her. B. No, because the man's promise to give his ex-wife $2,500 did not result in an economic benefit to him. C. Yes, because the amount of the ex-wife's reliance equaled the amount promised by the man. D. Yes, because the ex-wife took action in response to the man's promise.

Answer choice A is correct. Under the doctrine of promissory estoppel, a promise is binding if (i) the promisor should reasonably expect it to induce action or forbearance on the part of the promisee or a third person, (ii) the promise does induce such action or forbearance, and (iii) injustice can be avoided only by enforcement of the promise. Here, the ex-wife relied on the man's promise to give her $2,500. It is arguable whether the man could reasonably have expected his promise to send her money "for the house" would induce her to buy a big screen TV rather than paying the mortgage. Regardless, denying the ex-wife the money did not cause injustice to her because she should have put the money towards the mortgage, not an extravagance like a big screen TV. Answer choice B is incorrect because a promise may be enforceable under the doctrine of promissory estoppel even though carrying through on the promise will not result in an economic benefit to the promisor. Answer choice C is incorrect. Under the doctrine of promissory estoppel, there is no requirement that the promisee's degree of reliance as measured in monetary terms equal the amount promised. A promisee may be able to recover the amount that she has expended in reliance on a promise, such as preparatory costs, even though that amount is less than the promised amount. Answer choice D is incorrect. To be enforceable, the promise must induce the promisee or a third party to act. It is doubtful that the man reasonably expected that his promise would induce his ex-wife to take the action that she did—to contract to purchase a big screen TV. However, she can assert that the acquisition of the TV, as a household item, would be "for the house." Even though the ex-wife did take action in response to the man's promise, a court's refusal to enforce the promise would not be unjust.

A 17-year-old individual, who was one month shy of his 18th birthday, purchased a used car from its owner. The owner was aware of the individual's age, but, having known the individual and his family for many years, agreed to the sale. Under the terms of sale, which were agreed to by both parties orally, the owner immediately transferred title to the car to the individual, and the individual agreed to make monthly payments of $200 for two years. The day after turning 18 years old, the individual, while carelessly driving the car, was involved in an accident that resulted in serious injuries to the driver of the other car and totaled the individual's car. Upon learning about the accident later that day, the owner contacted the individual. The individual told the owner that he did not intend to pay for the car.The age of majority in the applicable jurisdiction is 18 years old.The owner of the car has filed a breach of contract complaint against the individual to recover the contract price of $4,800. Who is likely to be successful in this action? A. The owner, because the individual had anticipatorily repudiated his contractual obligation to pay for the car. B. The owner, because the car was totaled due to the individual's carelessness. C. The individual, because he told the owner that he did not intend to pay for the car. D. The individual, because the contract was oral.

Answer choice C is correct. When a contract is made by an infant, it is voidable by the infant but not by the other party. This means that the infant may either disaffirm (void) the contract and avoid any liability under it or choose to hold the adult party to the contract. The disaffirmance must be effectuated either before the infant reaches the age of majority or within a reasonable time thereafter. Here, the individual disaffirmed the contract for the sale of the car that he had entered into when he was 17 years old by refusing to make the payments required under the contract the day after he turned 18 years old. Consequently, it is likely that the individual will be successful in the action. Answer choice A is incorrect. Although it is true that the individual has anticipatorily repudiated his contractual obligation to make monthly payments of $200 for the next two years, the owner is not permitted to sue for the full amount currently because the only obligation of the individual is to pay money in installments. Answer choice B is incorrect. Although an individual who disaffirms a contract because of infancy is required to restore any benefits received under the contract if possible, an individual who is unable to do so may nevertheless disaffirm the contract, even though the inability to do so is due to the individual's own carelessness. Answer choice D is incorrect. A contract for the sale of goods for a price of $500 or more is subject to the Statute of Frauds. The UCC Statute of Frauds requires a memorandum of the sale that must (i) indicate that a contract has been made, (ii) identify the parties, (iii) contain a quantity term, and (iv) be signed by the party to be charged. Although the oral contract for the sale of the car for a total price of $4,800 ($200 x 24) is subject to the Statute of Frauds, and does not satisfy its requirements, the Statute of Frauds does not apply to a contract where the goods have been received and accepted. Here, the owner immediately transferred title to the car to the individual. In addition, a contract that cannot be performed within one year is subject to the Statute of Frauds. Even assuming that the contract prevented the purchaser from repaying the amount owed under the contract, full performance by either party to the contract will generally take the contract out of the Statute of Fraud under the one-year rule. Consequently, the Statute of Frauds, either under the sale of goods provision or the one-year rule, does not prevent the enforcement of this contract.

"A minor entered into a written contract with a grocery store in which the minor agreed to pay $75 a month in exchange for delivery of food items once a week to the minor's residence for a period of one year. For a couple of months, the minor paid the grocery store and the grocery store delivered the food each week. One month, the minor stopped paying the grocery store. The grocery store continued to deliver the food to the minor's residence despite the minor's nonpayment. At the end of two months of nonpayment, the grocery store contacted the minor and requested payment pursuant to the contract. In response, the minor communicated his disaffirmance of the contract. The grocery store then brought an action for breach of contract against the minor. It sought $150, the amount due under the contract. Is the minor liable on the contract?" A. Yes, but only for the reasonable value of the food and delivery service provided by the grocery store for two months. B. Yes, in the amount of $150. C. No, because the minor disaffirmed the contract. D. No, because the contract between the minor and the grocery store is void.

Answer choice A is correct. When a contract is made by an infant, it is voidable by the infant but not by the other party. This means that the infant may either disaffirm (void) the contract and avoid any liability under it or choose to hold the adult party to the contract. An exception to the infancy rule exists when the contract is based on necessities. When necessities are furnished to the infant, the infant must pay for them, but the recovery by the person furnishing the necessities is limited to the reasonable value of the services or goods (not the agreed upon price). In this case, the minor contracted with the grocery store to deliver necessities (food items) to the minor's residence once a week. Since the contract was for necessities, the minor must pay for them and cannot disaffirm the contract and avoid liability under it. However, the minor is only liable for the reasonable value of the goods and service provided by the grocery store, not the agreed upon price of $75 per month ($150 for the two months). Thus, answer choice B is incorrect. Answer choice C is incorrect. Since the contract was for necessities, the minor cannot disaffirm the contract and avoid liability. Answer choice D is incorrect. A contract entered into by a minor and an adult party is voidable by the minor; it is not void.

A wealthy family decided to have a family portrait painted. The mother contacted an artist and asked him to come to their home to paint the portrait. The mother and the artist entered into a valid written contract to paint the family's commissioned portrait for $15,000. The contract contained a clause by which the artist's payment was conditioned upon the family's satisfaction with the portrait. After the family finished sitting for the portrait and the artist completed his work, the artist revealed that he had painted the family's faces in pastel blue tones instead of using a pallet of flesh tones. Even though the portrait was of very high quality and would be objectively valued at approximately $15,000, the family disliked the aesthetic choice of color made by the artist and refused to accept or pay for the portrait. The artist subsequently sued to recover his fee under the contract.Is the artist likely to recover under the contract? A. No, because the artist's payment was conditioned upon the family's satisfaction. B. No, because the family did not accept or pay for the portrait. C. Yes, because the artist completed the portrait commissioned by the contract. D. Yes, because the portrait satisfied an objective standard of quality.

Answer choice A is correct. When the aesthetic taste of a party determines whether the other party's performance is satisfactory (e.g., painting a family portrait), satisfaction is determined under a subjective standard. Under this standard, if the party is honestly dissatisfied, even if the dissatisfaction is unreasonable, the condition has not been met. However, the party's dissatisfaction must be in good faith, or a claim of dissatisfaction can be a breach, such as when a party is asserting dissatisfaction merely to avoid its own contractual obligation. In this case, the condition of satisfaction was not met because the family disliked the artist's aesthetic choice of color. Per a clause included in the contract, the artist's payment was expressly conditioned upon the family's satisfaction with the portrait. Thus, the artist cannot recover under the contract. Answer choice B is incorrect. If the family had simply rejected the portrait without reference to the condition of satisfaction, that would likely be a breach of contract. Additionally, even if they had accepted the portrait while asserting in good faith that the condition of satisfaction was not met, the artist would not have been able to recover under the contract. Instead, he would have to bring an action for unjust enrichment. Answer choice C is incorrect because the artist's right to payment under the contract was conditioned upon the family's satisfaction with the portrait. Because the family was not satisfied, the artist cannot recover under the contract. Answer choice D is incorrect because, as discussed above, satisfaction is determined under a subjective standard when a condition involves aesthetic taste.

A tennis instructor gives group lessons for adult beginners at a tennis court in a private club. A novice adult player showed up at the court to participate in the drills conducted by the instructor. The instructor gave the novice instruction on her tennis game for that day and several more days. The lessons cost $50 per hour. The novice refused to pay the instructor. In a breach of contract action to recover the lesson fees, will the instructor prevail? A. Yes, because the bargain, if expressly made, would have been supported by consideration. B. Yes, because the conduct of the novice manifested assent to the bargain. C. No, because the instructor's only recovery would be in quantum meruit. D. No, because there was no formal offer and acceptance.

Answer choice B is correct because an implied-in-fact contract exists. When a person's assent to an offer is inferred from the person's conduct and the person has reason to know his conduct may be construed as assent, an implied-in-fact contract exists. Here, the novice's participation in the drills implied such consent. Answer choice A is incorrect because although implied-in-fact contracts need to be supported by consideration, this is not the definitive issue relating to the bargain's enforcement. Answer choice C is incorrect because this is an implied-in-fact contract, and therefore breach of contract remedies would be available; quantum meruit remedies are available in quasi-contract actions, which this is not. Answer choice D is incorrect because even though there was no formal offer and acceptance, an implied-in-fact contract does not require a formal offer and acceptance.

A biker agreed to buy a vintage motorcycle from a mechanic for $10,000, once the mechanic finished the restoration. The agreement provided that the biker would pick up and pay for the motorcycle on May 15. To protect the motorcycle from damage in his busy shop, the mechanic rented a garage to store the motorcycle while he restored it. After storage fees and the costs of the restoration materials, the mechanic expected a$4,000 profit from the contract. On May 15 when the biker came to pick up the motorcycle, even though the motorcycle was well-restored, the biker, for personal financial reasons, refused to pay for the motorcycle.The mechanic stored the motorcycle for another six months while he looked for another buyer, paying $900 in storage fees. He then sold the motorcycle for $7,000 to another buyer. Which of the following is the highest proper measure of the mechanic's damages? A. $3,000, the difference between the contract price and the price obtained from a proper resale of the motorcycle. B. $3,900, the difference between the contract price and the price obtained from a proper resale of the motorcycle, plus the cost of six months of storage. C. $4,000, the mechanic's expected profit for the contract. D. $10,000, the contract price of the motorcycle.

Answer choice B is correct. A seller can recover the difference between the contract price and resale price when a buyer has wrongfully rejected the goods. In addition, a seller can recover incidental damages (including storage and shipping costs) when a buyer wrongfully rejects goods. Answer choice A is incorrect because it does not include the seller's incidental damages. Answer choice C is incorrect because only a volume seller of goods can recover the expected profit of a sale. Answer choice D is incorrect because an action for contract price is only available under the following three circumstances (i) when the goods have been accepted by the buyer; (ii) when the risk of loss has passed to the buyer and, within a commercially reasonable time afterwards, the goods are lost or damaged; or (iii) when the seller is unable, after making a reasonable effort, to resell the goods that have been rejected or returned by the buyer. None of these circumstances are present in this question.

In a telephone conversation, a real estate investor promised to pay a home owner $500,000 for the owner's two-story house. The investor gave the owner $100,000 and told the owner he was going to have a crew come over to do renovation work on the house. Over the next three weeks, the investor's crew gutted the house's kitchen and replaced it with high-end cabinets and appliances, laid new flooring, installed new insulation, and rewired the electrical system in the entire house. When the agreed-upon date arrived for closing, the owner told the investor that he had changed his mind and would not sell the house. The investor sued for enforcement of the real estate sales contract. Will he likely prevail? A. Yes, because the owner agreed to sell the house to the investor. B. Yes, because the investor had partially performed the contract. C. No, because the agreement was not in writing. D. No, because the investor only paid the owner $100,000 before he started renovating the house.

Answer choice B is correct. Although a promise to transfer or receive any interest in real property is within the Statute of Frauds and therefore must be in writing and signed by the party against whom enforcement is sought, subsequent acts by either party that show the existence of the contract may make it enforceable. Most jurisdictions require two of the three following acts to establish sufficient part performance: (i) payment of all or part of the purchase price; (ii) possession by the purchaser; or (iii) substantial improvement of the property by the purchaser. Here, the investor has at least met requirements (i) and (iii) and, depending on the court's interpretation, possibly (ii). The oral agreement is therefore likely enforceable. Answer choice C is therefore incorrect. Answer choice D is incorrect because partial payment is sufficient to meet requirement (i). Answer choice A is incorrect because an oral agreement to sell real estate is not usually enforceable under the Statute of Frauds; the partial performance here is an exception to the rule.

The owner of a mine and a wealthy friend were having lunch in a restaurant. The owner had operated the mine as a successful business for a number of years and planned to continue to do so indefinitely, but was always boasting to friends that he could sell the business for far more than its asset value. On this occasion, the owner stated to the friend that he would sell her his business for $15 million, even though its assets were only worth $10 million. The friend responded "If that's your best offer, I can't accept it." The owner then wrote on a napkin, "I, Owner, hereby offer to sell my mining business to Friend, for $15 million." The friend took the napkin, wrote, "I accept your offer," then signed her name and pocketed the napkin. Nothing more was said about the sale. The next day, the friend, upon learning that a valuable vein had been discovered in the mine that day, contacted the owner. The owner refused to transfer the mine to his friend. In a breach of contract action by the friend against the mine owner, which of the following is the owner's best defense to this action? A. The owner did not intend to sell his mining business. B. The friend did not reasonably believe that the owner intended to sell his mining business. C. The writing by the owner was not sufficiently definite to constitute an offer. D. The friend had rejected the owner's offer to sell his mining business for $15 million.

Answer choice B is correct. Although a stranger may have considered the owner's actions an objective manifestation of intent to enter into a contract, the friend, based on the owner's past behavior, could not reasonably believe that the owner intended to sell the business. Answer choice A is incorrect because the subjective intent of the owner would not prevent the formation of a contract where the owner's actions constituted an objective manifestation to enter into a contract. Answer choice C is incorrect because the writing contained the key terms (i.e., subject, parties, price) and thus was sufficiently definite to constitute an offer. Answer choice D is incorrect because, although the friend did initially reject the owner's purported offer to sell the business for $15 million, the owner renewed the purported offer after the rejection by writing it on the napkin.

A corporation decided to hold an annual retreat for a long weekend during the summer. The corporation entered into a written agreement with a contractor for $8,000 to build a large stage on which the president, CEO, and other corporate officers could deliver speeches and present different programs. The stage was to contain all of the sophisticated technology necessary for innovative programming. The contractor was also responsible for installing a large tent to cover the area next to the stage, in case of bad weather. The agreement mandated that the work had to be completed by 6:00 p.m. on the day before the annual retreat was set to begin. The contractor was on vacation the week leading up to the annual retreat and decided to spend an extra day away. Two days before the corporation's annual retreat, the contractor finally began working on the project. Due to the time constraints, he realized that he would not be able to complete the project without extra help at an additional cost of $1,000. The corporation agreed to the additional cost because it had no other option and needed the project completed on time. After the contractor timely completed the project, the corporation paid him $8,000 but refused to pay the additional $1,000. Can the contractor recover the additional $1,000 from the corporation? A. No, because the agreement to pay the additional cost was not in writing. B. No, because there was no consideration for the corporation's promise to pay the additional amount. C. Yes, because the corporation agreed and the contractor completed performance on time. D. Yes, because the unforeseen difficulties make this modification enforceable.

Answer choice B is correct. At common law, modification of an existing contract must be supported by consideration. Here, the contractor was already obligated to build the large stage and install the tent and has suffered no additional legal detriment that could be considered consideration. The corporation did not receive any consideration from the contractor for its promise to pay the additional $1,000. A modification of a contract can also be enforced when there are unforeseen difficulties, and one of the parties agrees to compensate the other when the difficulties are discovered if those difficulties would make performance impracticable. However, performance is not made impracticable here because the contractor willingly extended his vacation. Therefore, the contractor cannot recover the additional $1,000. Answer choice A is incorrect because the modification to this services contract did not need to be in writing to be enforceable. However, consideration was required in order for the modification to be enforced. Answer choice C is incorrect. Although the corporation promised to pay the additional amount, there was no consideration to support this promise and thus it is not enforceable. The fact that the contractor completed performance on time does not entitle him to recover the additional $1,000. The contractor was already obligated to complete performance on time in exchange for $8,000. Answer choice D is incorrect. A modification of a contract can be enforced when there are unforeseen difficulties, and one of the parties agrees to compensate the other when the difficulties are discovered if those difficulties would make performance impracticable. For the defense of impracticability to be available, an unforeseeable event must occur, nonoccurrence of the event must be a basic assumption on which the contract was made, and the party seeking discharge must not be at fault. Non-extraordinary increases in the cost of performance are not on their own impracticable. In this case, there was not an unforeseeable event. It is foreseeable that if a contractor starts working on a project late he might need to hire extra help to complete the work on time. Moreover, even if the contractor believed he could complete the work in two days and thus the event could be considered "unforeseeable," the contractor was at fault by taking an extra vacation day.

On May 15, a toy manufacturer entered into a signed, written agreement with a distributor for the sale of a new line of toys. The agreement stated that the manufacturer would immediately ship 1,000 of the new toys to the distributor at a price of $4 per unit. Under the agreement, until June 15, the distributor could only sell the toys to certain retailers listed in the agreement in order to create a high demand for the new toys. Provided that the distributor complied with this requirement, the toy manufacturer would ship an additional 1,000 units for $2 per unit on June 15, and the distributor could then sell the toys to any retailers it chose. On June 1, the toy manufacturer learned that the distributor had sold some of the toys to one retailer not specifically listed in the agreement. On June 16, the distributor contacted the manufacturer to arrange for the shipment of the additional 1,000 units. The manufacturer refused to ship any additional units to the distributor. If the distributor sues the manufacturer for breach of contract, will it prevail? A. No, because the distributor acted in bad faith. B. No, because the shipment of the additional units was conditioned on the distributor only selling to listed retailers. C. Yes, because the distributor substantially complied with the condition regarding sale to listed retailers. D. Yes, because the manufacturer refused to ship the additional units as required under the agreement.

Answer choice B is correct. Express conditions are specified in the contract. Express conditions must be complied with fully unless excused; substantial compliance will not suffice. Here, the agreement between the toy manufacturer and the distributor contained an express condition requiring the distributor to only sell to the retailers specifically listed in the agreement. The shipment of the additional 1,000 units was conditioned on the distributor only selling to those particular retailers. The distributor failed to meet this condition because it sold some of the toys to a retailer not listed in the agreement. Therefore, the manufacturer was not obligated to deliver the additional 1,000 units. Answer choice A is incorrect. The UCC implies good faith as a contract term in all contracts. Although the distributor failed to comply with the express condition in the agreement, it is unlikely that the distributor's actions amounted to bad faith. Answer choice C is incorrect. The doctrine of substantial performance does not apply either to express conditions or to contracts for the sale of goods under the UCC. Answer choice D is incorrect. The manufacturer was not obligated to perform under the agreement because the distributor failed to comply with the express condition in the agreement.

A shoe manufacturer contends that the owner of a shoe store called and ordered 50 pairs of Oxford-style dress shoes at $100 per pair to be shipped within three weeks and that the manufacturer's representative immediately accepted this order. The manufacturer promptly sent the owner a signed, written acknowledgment of the alleged order that reflected the manufacturer as seller and the shoe store owner as buyer, as well as the number and style of shoes, but that did not indicate the price of the shoes. The owner admits to receiving the acknowledgment the following day and taking no action regarding it. Two weeks later, the owner received a shipment of 50 pairs of Oxford-style dress shoes. The owner immediately called the manufacturer and asserted that he had never ordered the shoes. Will the Statute of Frauds prevent the manufacturer from enforcing this contract against the owner? A. No, because an oral contract made between merchants is enforceable. B. No, because the owner received and did not respond to the written acknowledgment in a timely manner. C. Yes, because the acknowledgment did not indicate the price of the shoes. D. Yes, because the price of the shoes exceeds the $500 threshold of the Statute of Frauds.

Answer choice B is correct. Generally, a contract that falls within the Statute of Frauds is unenforceable unless evidenced by a writing. The writing must (i) be signed by the party to be charged and (ii) contain the essential elements of the deal. A contract for the sale of goods for a price of at least $500 falls with the Statute of Frauds. Consequently, the purported order of 50 pairs of shoes at $100 per pair is subject to the Statute of Frauds. Although the acknowledgment sent by the manufacturer to the owner is otherwise sufficient to satisfy the Statute of Frauds, it was not signed by the owner. However, if both parties are merchants and a memorandum sufficient against one party is sent to the other party, who has reason to know its contents, and the receiving party does not object in writing within 10 days, then the contract is enforceable against the receiving party even though he has not signed it. Here, this merchant's exception applies to the owner who received the acknowledgment of his order and did not reply for more than 10 days after receiving it. Answer choice A is incorrect. Although a memorandum of an oral agreement sent by one merchant may permit the enforcement of the agreement against another merchant, there is no blanket exception to the Statute of Frauds that permits enforcement of an oral agreement between two merchants. Answer choice C is incorrect. Under the Statute of Frauds for the sale of goods, the price of the goods need not be included in the contract. If the price is omitted, the parties can present evidence of the agreed-upon price or (if that fails) the market price. Therefore, the failure of the acknowledgment to reflect the price of the shoes does not prevent the acknowledgment from satisfying the merchant's exception to the Statute of Frauds. Answer choice D is incorrect. Although the Statute of Frauds for a sale of goods does apply to this purported transaction and was not satisfied because the owner did not sign the acknowledgment, the acknowledgment sent by the manufacturer satisfies an exception to the Statute of Frauds when both parties to the oral agreement are merchants.

A customer visited several area car dealerships, looking for a family car. One local car dealer named a price for the car that the customer wanted, which the customer asked be put in writing. The dealer wrote down the car's unique vehicle identification number, the price, the date, and the statement, "Firm offer for 30 days from today's date, provided car is in stock." The dealer signed the document. Twenty-nine days later, the customer returned to the dealer. The dealer admitted that the car was still in stock, but told the customer that it would now cost $500 more. The customer replied that he was ready, willing and able to buy the car, but only at the lower price. Does the customer's statement constitute acceptance of the dealer's lower price for the car? A. Yes, because the customer was not a merchant. B. Yes, because the dealer was a merchant. C. No, because the offer was subject to a condition and was therefore not a firm offer. D. No, because the dealer withdrew the offer of a lower price before the customer accepted it.

Answer choice B is correct. Generally, unless consideration is given to keep an offer open, the offeror can withdraw an offer at any time prior to its acceptance by the offeree, even an offer that the offeror states is irrevocable. However, a firm offer made by the offeror in a signed writing is irrevocable for the stated time period, where such period is three months or less, provided that the offeror is a merchant. For purposes of this rule, a merchant includes not only a person who regularly deals in the type of goods involved in the transaction or otherwise by his occupation holds himself out as having knowledge or skill peculiar to the practices or goods involved in the transaction, but also any businessperson when the transaction is of a commercial nature. Here, the car dealer is a merchant who regularly deals in the type of goods involved in the transaction (here, cars). Therefore, the dealer's attempt to revoke his offer was unsuccessful. By making the customer a firm offer in a signed writing, that offer was irrevocable and the customer could and did accept it within the 30-day period fixed by the offer. Answer choice A is incorrect because the firm offer rule applies to any sale of goods by a merchant, regardless of whether the buyer is or is not a merchant. Answer choice C is incorrect because any offer, including a firm offer, may be subject to a condition. The condition serves as a limitation on the terms of the offer, but does not prevent the offer from being a firm offer. Moreover, in this case, the condition was satisfied, since the car had not been sold. Answer choice D is incorrect because, as noted with respect to answer choice B, the dealer had given the customer an irrevocable firm offer for 30 days.

A hotelier owned a hotel and the surrounding land. A portion of the land immediately behind the hotel was unused, and the hotelier wanted to develop it and install an in-ground pool. She contacted a general contractor in January, and the parties agreed that the contractor would build out a large terrace complete with a fifty-foot in-ground pool. They agreed to a price $400,000, with the contract to be completed before March 15, payable upon completion of the work. The hotelier stressed to the contractor the importance of completing the work before March 15 because that was when her vacation season began and she planned a special advertisement of the pool terrace to boost sales. The hotelier paid the contractor the $150,000 initial payment, but the contractor abandoned the job in late January without performing any work on the site. The hotelier hired a second contractor in February, who charged her $550,000 to complete the job. Despite the rush, the terrace was not ready until April 15, and the hotelier lost $70,000 in net profit due to documented reservation cancellations because of the unfinished construction. What is the most a court could properly award the hotelier? A. $550,000 B. $370,000 C. $220,000 D. $150,000

Answer choice B is correct. In construction contracts, the general measure of expectation damages for failing to perform is the difference between the contract price and the cost of completion by another builder. Here, that difference is the $550,000 cost to hire the replacement contractor minus the $400,000 contract price, or $150,000. Further, consequential damages may be awarded for reasonably foreseeable losses to a nonbreaching party that go beyond expectation damages, such as loss of profits. Damages are recoverable if they were the natural and probable consequence of breach, or if they were in the contemplation of the parties at the time the contract was made, and may be calculated with reasonable certainty. Here, lost profits due to room cancellations is a natural and probable consequence of ongoing construction at the hotel because guests may have specifically wanted to stay in a hotel with a pool or else avoided a hotel that had ongoing construction on the premises. Further, the hotelier had specifically told the contractor that she planned to specifically advertise the pool terrace to boost sales. Additionally, the hotelier had detailed documentation to support her claim of lost profits. Therefore, a court could award her the $70,000 in lost profits. Finally, the hotelier would be able to recover the $150,000 that she originally paid to the first contractor as a deposit. Therefore, the correct measure of damages would be the $70,000 in lost profits, (consequential damages) plus the $150,000 in expectation damages, plus the $150,000 deposit, for a total of $370,000. Answer choice A is incorrect because this assumes the hotelier would receive the entire $550,000 replacement contract price (an incorrect measure of expectation damages). It also ignores the $70,000 in consequential damages and the deposit. Answer choice C is incorrect because it correctly measures expectation damages and consequential damages, but fails to take into account the deposit. Answer choice D is incorrect because it incorrectly omits the $70,000 in consequential damages and the deposit that the court could award.

A retailer received a written firm offer signed by a supplier. The offer committed the supplier to providing the retailer with up to 10,000 tubes of toothpaste over the next 45 days at $1 a tube. Thirty days later, the supplier informed the retailer that the price per tube of toothpaste would be $1.10. The next day the retailer ordered 6,000 tubes of toothpaste from the supplier, which the supplier promptly shipped. Sixty days after the receipt of the offer, the retailer ordered another 4,000 tubes of toothpaste, which the supplier also promptly shipped. What price is the supplier permitted to charge the retailer for the toothpaste? A. $10,000 (10,000 * $1), because the supplier's firm offer was effective for three months regardless of its terms. B. $10,400 ((6,000 * $1) + (4,000 * $1.10)), because the supplier's firm offer was effective for only 45 days. C. $11,000 (10,000 * $1.10), because the firm offer rule does not apply where the buyer is a merchant. D. $11,000 (10,000 * $1.10), because the supplier informed the retailer that the price was increased to $1.10 before the retailer's placement of either order.

Answer choice B is correct. The supplier's written and signed firm offer was effective and irrevocable for 45 days, even though no consideration was provided by the retailer to the supplier for making the firm offer. Consequently, the price for the first shipment of 6,000 tubes of toothpaste was $1 per tube. Since the firm offer had expired prior to the placement of the second order, the supplier's price of $1.10 per tube was in effect with regard to this order. Answer choice A is incorrect because, while a firm offer made by a merchant may be effective up to three months, the merchant offeror, as with any offeror, is master of the offer. Consequently, the merchant offeror could set the time period for the firm offer at 45 days. Answer choice C is incorrect because the firm offer rule applies, even where the buyer is a merchant, as long as the seller is a merchant. Answer choice D is incorrect because, as noted with regard to answer choice B, the firm offer made by the supplier applies to the first order placed by the retailer because such offer was irrevocable.

A merchant and manufacturer entered into a contract for the sale of a large grandfather clock built by the manufacturer. The manufacturer already had a number of grandfather clocks built, each with slight aesthetic differences, and the contract provided for the sale of a particular clock identified by serial number. The contract specified that the manufacturer would ship the clock by a third-party carrier. However, the contract did not specify either who was to pay the costs of carriage or the place of tender for the clock. The manufacturer shipped the clock identified in the contract to the merchant via a carrier pursuant to a proper contract for its carriage. The carrier delivered the clock to the merchant and the merchant verified that the clock delivered was the one specified in the contract. Under these facts, when did risk of loss pass to the merchant? A. When the contract was made. B. When the clock was delivered to the carrier and a proper contract for its carriage was made. C. When the clock was unloaded on the merchant's premises by the carrier. D. When the merchant verified that the clock delivered was the clock specified in the contract.

Answer choice B is correct. Under UCC § 2-509(1), a contract that requires the seller to ship goods to the buyer by a third-party carrier is either a shipment or a destination contract. When the contract requires shipment by a third-party carrier, a shipment contract is presumed unless the contract indicates otherwise (i.e., unless it specifies delivery to a particular place). Because this is presumed to be a shipment contract, the risk of loss would pass from the manufacturer to the merchant when the manufacturer duly delivered the clock to the third-party carrier. Therefore, answer choice B is correct. Answer choice A is incorrect because UCC § 2-509(1)(a) and (b) do not pass the risk of loss from the seller to the buyer at the time of contract formation. Answer choice C is incorrect because this is not a destination contract; the contract did not specify delivery to a particular place. Answer choice D is incorrect because, although UCC § 2-510 places the risk of their loss on the seller when a tender or delivery of goods so fails to conform to the contract as to give a right of rejection, since the manufacturer shipped the clock identified in the contract to the merchant, the merchant's act of identifying the clock as conforming to the contract does not alter the passing of risk to the merchant when the clock was delivered to the carrier and proper contract for its carriage was made.

A wholesaler of bicycle chains sent a retailer the following fax on December 1: "Because of your continued loyalty as a customer, I am prepared to sell you up to 1,000 units of Bicycle Chain Model D at $7.50 per unit, a 25% discount off our original $10.00 price. This offer will remain open for 7 days." The fax lacked a full, handwritten signature, but was on the wholesaler's letterhead and had been initialed by the wholesaler's head of sales. On December 4, the wholesaler's head of sales called the retailer and informed the retailer that he had decided to revoke his December 1 offer. On December 5, the retailer placed an order for 1,000 bicycle chains, stating that he would pay the discounted price of $7.50 per unit. What is the correct value of the order placed by the retailer? A. $7,500, because the wholesaler's revocation was not in writing. B. $7,500, because the wholesaler was bound to keep the offer open for 7 days. C. $10,000, because the offer was not signed by the wholesaler. D. $10,000, because the retailer did not provide consideration to hold the offer open.

Answer choice B is correct. Under the UCC's firm offer rule, an offer to buy or sell goods is irrevocable if the offeror is a merchant, there is an assurance that the offer is to remain open, and the assurance is contained in a signed writing from the offeror. No consideration by the offeree is needed to keep the offer open. Here, all three conditions are satisfied (note that letterhead and an agent's initials suffices as a "signature"), and the wholesaler's offer was irrevocable until the offer period expired. The retailer's order was within the 7-day window, and thus the order for 1,000 units is priced at $7.50 each, or $7,500 total. Answer choice A is incorrect because the wholesaler was bound by the UCC firm offer rule, and his offer was irrevocable regardless of the method of his attempted revocation. Answer choice C is incorrect because the initials of the head of sales on the letterhead are sufficient to constitute a signed writing under the UCC. Answer choice D is incorrect because, under the UCC's firm offer rule, no consideration was required to hold the offer open.

A widget manufacturer contracted in writing to deliver to a warehouse 500 tin widgets on November 1. In October, the price of tin rose, and the manufacturer could not obtain tin to fill the warehouse's order without losing money on the transaction. However, the manufacturer still had leftover copper from a previous order that had been canceled. On November 1, the manufacturer delivered 500 copper widgets to the warehouse, explaining in good faith that it had no tin to work with and that the copper widgets were of a higher quality. The manufacturer then asked the warehouse to pay the contract price they had agreed upon for the lower quality tin widgets. The warehouse refused to accept the copper widgets and refused to pay the manufacturer any money. Which of the following most accurately describes the legal relationship between the warehouse and the manufacturer? A. The warehouse must give the manufacturer an opportunity to cure the breach before bringing a claim against the manufacturer. B. The warehouse can immediately bring a valid breach of contract claim against the manufacturer for its failure to deliver tin widgets. C. The manufacturer's duty under its contract with the warehouse was excused by the unavailability of tin. D. The manufacturer can immediately bring a valid breach of contract claim against the warehouse for rejecting the copper widgets.

Answer choice B is correct. Under the UCC, the basic obligations of a seller are to transfer ownership of the goods to the buyer and to tender goods conforming to the warranty obligations. The UCC requires "perfect tender," and substantial performance will not suffice except for installment contracts, or when the parties agree that it applies. Thus, under the UCC's perfect tender rule, the manufacturer has breached the contract with the warehouse, and the warehouse can immediately bring an action against the manufacturer. Answer choice A is incorrect because the warehouse has no obligation to allow the manufacturer to cure the breach of this contract before bringing an action, as the time for performance has elapsed and there are no facts to indicate the manufacturer had reasonable grounds to believe that the warehouse would accept despite the nonconformity. Additionally, to have the right to cure, the seller must give notice of the intent to cure and make a new tender of conforming goods. Here, the manufacturer has given no such notice. Answer choice C is incorrect because non-extraordinary increases in the cost of performance are not sufficient to excuse performance. Answer choice D is incorrect because the warehouse was entitled to reject this nonconforming shipment of copper widgets by the manufacturer, despite their higher quality. Therefore, the manufacturer does not have a claim against the warehouse for breaching its duty to purchase the widgets.

An amateur historian sought to purchase a house and its surrounding land because he believed, based on his own research, that the property had been the site of a key battle in the U.S. Civil War. The historian discussed this fact with the owner of the property as they inspected the land, telling the owner of his plans to dismantle the house and landscape the land to rent it to a local historical reenactment group who wanted to reenact the battle. The owner of the property had never heard of the battle, and although he listened with interest, he did nothing to expressly confirm or deny the historian's belief. The historian entered into a written contract with the owner for the property. A few days prior to the closing date, the historian contacted the historical reenactment group to offer the land as a site for a reenactment, but the group correctly explained to the historian that he was mistaken—the battle had actually taken place two miles north of his property. If the historian sues the owner to void the contract, is he likely to prevail? A. No, because the fact that the battle did not occur on the property did not materially affect the bargain. B. No, because the historian bore the risk of the mistake regarding where the Civil War battle actually took place. C. Yes, because the historian's purpose in entering into the contract was frustrated. D. Yes, because the owner's failure to confirm or deny the historian's belief constituted a breach of the duty of good faith.

Answer choice B is correct. When only one of the parties is mistaken as to an essential element of the contract, either party can generally enforce the contract on its terms. However, the mistaken party can void the contract if the he did not bear the risk of the mistake. Here, the historian bore the risk of the unilateral mistake regarding the actual location of the battle, because there is no indication that the risk of that mistake was assumed by the owner. Answer choice A is incorrect because, although the association of the property with Civil War history did not affect the condition of the property, the historian sought the property for its alleged historical significance, making it material to the bargain. Answer choice C is incorrect because the doctrine of frustration of purpose applies when unexpected events arise that destroy one party's basic assumption of the contract. Here, although the owner knew that the historian wanted to buy the land so that he could rent it out to a local historical reenactment group, his ability to do so was not a basic assumption of the contract. In addition, frustration of purpose requires that the frustration be so severe that it is not within the assumed risks inherent under the contract. Here, the possibility that the historian might not be able to rent the property to the reenactment group was within the assumed risks inherent to the contract. Answer choice D is incorrect because the duty of good faith generally does not impose an affirmative duty to act, but instead simply requires honesty in fact. Here, the owner did not create the historian's erroneous belief, and thus he was under no obligation to investigate or confirm the historian's claims.

A daughter wanted to go to law school after she graduated from college. She took out a loan of $150,000, intending to use it to pay for her law school tuition. When she told her father about the loan and her plan, the father told her, "I think that is a very wise career choice, and I want to support you. If you go to the same law school I attended, when your loans go into repayment in three years, I will handle it." The daughter did not reveal that she had been accepted by the same law school as her father and had already enrolled there. Instead, she thanked him and went to the law school. After two and a half years, the daughter decided she did not want to practice law. She dropped out and used the remainder of her loan to travel the world. Her loan went into repayment three years after the father made his promise. She asked her father to start making the loan payments on her behalf. The father refused. The daughter has brought an action against her father to enforce his promise to repay the daughter's loan. Is the daughter likely to succeed in her action? A. No, because the agreement could not be performed within a year. B. No, because there was no consideration provided to support the father's promise. C. Yes, because the daughter fully performed by attending the same law school as her father. D. Yes, under the doctrine of promissory estoppel.

Answer choice C is correct. A contract that cannot be performed within one year from its making because of the constraints of the terms of the agreement falls within the Statute of Frauds and is unenforceable unless made in writing. Here, the father's promise could not have been completed within a year of its making because he specifically promised to pay the loan in three years. However, full performance by either party to the contract will generally take the contract out of the Statute of Frauds. Here, the daughter fully performed by attending the same law school as her father. Therefore, the father's promise is enforceable despite the lack of a writing that satisfies the Statute of Frauds. Answer choice A is incorrect because although the agreement could not be performed within one year, as the father promised to repay the loans in three years when they became due, and would therefore fall within the Statute of Frauds, the daughter's performance takes the contract out of the Statute of Frauds. Answer choice B is incorrect because the daughter's attendance at the same law school her father attended was sufficient consideration to support his promise. Answer choice D is incorrect because the daughter did not take any action in reliance on her father's promise to pay her debts. The daughter had taken out the $150,000 loan prior to informing her father about her plan to go to law school, and had already enrolled at the law school he attended. Thus, she did not rely on her father's promise in obtaining the $150,000 loan or in enrolling in this particular law school. Accordingly, the doctrine of promissory estoppel does not apply.

Two roommates lived together in a modest apartment for many years, during which time the female roommate purchased many items of furniture for the apartment. These items were worth roughly $10,000. When the female roommate decided to relocate across the country for a large promotion, she told the male roommate he could have all of the furniture when she left at the end of the month, as he had a much lower income. He wrote a long thank-you note telling her he was grateful. The next day at work, before the female roommate read the note, she learned that her promotion fell through and she decided to stay in the apartment.At the end of the month, can the male roommate enforce the female's promise to give him the furniture? A. Yes, because he accepted the offer of the furniture through the thank-you note. B. Yes, because he acquired full ownership of the property upon acceptance. C. No, because the male roommate did not bargain for a legal detriment in exchange for the promise. D. No, because the female roommate had not yet read the note regarding acceptance.

Answer choice C is correct. A promise to make a gift does not involve bargained-for consideration and is therefore unenforceable. In order to be enforceable under the majority rule, the promisee must suffer a bargained-for detriment. Here, the female roommate promised to give the male roommate the furniture as a gift. He gave her nothing in return, and no other consideration was provided. The promise is not enforceable because under the terms of the female roommate's promise, the male roommate would receive the furniture without any bargained-for detriment. Therefore, the male roommate cannot enforce the promise. Answer choice A is incorrect because the promise to give the man the furniture was unenforceable, even if he attempted to accept it. Furthermore, the fact that it was a thank-you note supports that the promise was a gift. Answer choice B is incorrect because the promise was unenforceable due to lack of consideration. The male roommate did not acquire full ownership when he wrote his thank you letter. Answer choice D is incorrect because, even if the woman had read the thank-you note, her promise was to make a gift and was therefore unenforceable unless the male roommate suffered a bargained-for legal detriment.

A store owner and a builder entered into a contract for the construction of an addition to the owner's store for $500,000. The builder calculated that he was to earn a profit of $20,000 for the job. A month later, the builder had spent $80,000 on labor and materials, including $15,000 on uninstalled windows. The reasonable market value of the labor and materials provided at that time was $50,000. At this point, the owner told the builder to stop working because the owner could no longer pay for the addition. The builder was able to sell the windows to a contractor for $10,000.In an action by the builder against store owner, which of the following is the highest amount of damages the builder may recover? A. $50,000, the reasonable value of the services the builder provided to the owner. B. $70,000, the builder's construction costs. C. $90,000, the builder's construction costs of $80,000 plus the $20,000 profit minus the $10,000 saved by selling the windows. D. $100,000, the builder's construction costs of $80,000 plus the $20,000 profit.

Answer choice C is correct. A recovery of $90,000 represents the builder's expectation measure of recovery and gives him the benefit of the bargain. The general measure of damages for the owner's failure to pay the contract price, in whole or in part, is the profits that the builder would have earned ($20,000), plus any costs incurred by the builder ($80,000), less the amount of any payments made by the owner to the contractor ($0) and any materials purchased by the contractor that are used by the contractor on another job or sold to a third party ($10,000). This amount would place the builder in the position he would have been in but for the breach. It is also the greatest amount the builder is able to recover. Answer choice A is incorrect because it is a restitutionary measure of damages that will not allow for damages related to benefit of the bargain. Answer choice B is incorrect. Not only does answer choice B also fail to consider the benefit of the bargain, but answer choice B would be the Builder's reliance damages, and there are no circumstances in this fact pattern that would cause the builder to seek reliance damages over expectation damages. Answer D is incorrect because it fails to subtract the loss avoided by the builder's sale of the windows to the contractor.

A produce wholesaler owed a sizeable debt that was past due. The lender offered, in exchange for a small parcel of land owned by the wholesaler, to reduce the amount of the debt by an amount equal to half of the fair market value of the land; the wholesaler refused. In order to compel the wholesaler to sell the land, the lender threatened to sue the wholesaler for the outstanding amount of the debt and to immediately attach a major shipment of produce to the wholesaler's most important and newly acquired client, although the wholesaler had sufficient other assets that could be attached. The wholesaler, feeling he had no choice since he did not have the cash on hand to pay off the debt, sold the land to the lender. The wholesaler subsequently sued the lender to void the sale. Will the wholesaler be able to void the sale? A. No, because the lender's threat was economic. B. No, because the wholesaler owed the amount that the lender threatened to collect. C. Yes, because the wholesaler sold the land under duress. D. Yes, because the wholesaler had been threatened with litigation.

Answer choice C is correct. Duress is an improper threat that deprives a party of meaningful choice. While the lender's threat of initiating a civil action to recover the debt was not improper since the debt was past due, the lender's use of the attachment process was improper. The lender was using the process, not to ensure the recovery of the debt, but to force the wholesaler to sell the land to lender at an unfair price since the wholesaler had sufficient other assets that the lender could have attached in order to protect himself from the possibility of non-payment. The wholesaler lacked a reasonable alternative to succumbing to the threat. Answer choice A is incorrect because, while generally an economic threat is not sufficient to give rise to duress, such a threat can constitute duress where the threat is improper and the threatened party is deprived of meaningful choice. Answer choice B is incorrect because, while the lender's threat of initiating a civil action to recover the debt was not improper since the debt was past due, the lender's use of the attachment process was improper. Answer choice D is incorrect because the threat of civil litigation is generally not improper. It is only improper if the lawsuit is pursued in bad faith.

In February, a vendor and a manufacturer entered a written contract under which the manufacturer would supply the vendor with a shipment of widgets each month for sale in the vendor's business. The contract provided that the manufacturer would "deliver 100 units of widgets on the last business day of each month until the end of the year." The manufacturer made satisfactory deliveries for seven months, and the vendor accepted each shipment. However, the vendor rejected the shipment delivered at the end of September. The manufacturer has sued the vendor for breach of contract. The vendor seeks to admit evidence from the pre-contract negotiations between the parties that the term "year" in the contract was understood by both parties to mean the end of the vendor's "fiscal year" in August, and therefore the vendor was under no further obligation to accept shipments from the manufacturer. The court has found that the term "year" in the contract is ambiguous.If the court finds that the February written contract is completely integrated, is the vendor's proffered evidence admissible? A. No, because the "four-corners" rule requires that the objective definitions of ambiguous contract terms control the meaning of the contract. B. No, because the parol evidence rule makes this evidence inadmissible. C. Yes, because the court has found that the term "year" is ambiguous. D. Yes, because the UCC permits the admission of this evidence even if the term was unambiguous.

Answer choice C is correct. Evidence may be admitted for the purpose of interpreting or clarifying an ambiguity in the agreement. This can include evidence of trade usage or even local custom to show that a particular word or phrase had a particular meaning. Here, because the court has found that the term "year" is ambiguous, the court will admit the vendor's evidence that the word "year" should be interpreted to mean "fiscal year." Answer choice A is incorrect because the common-law "four-corners" rule applies when extrinsic evidence is being offered to prove a contract term, not to interpret contract language. Answer choice B is incorrect because the parol evidence rule does not apply when extrinsic evidence is offered to determine the correct interpretation of a word in a contract. Answer choice D is incorrect. Even if the terms of a written contract for the sale of goods appear to be unambiguous, a party may explain or supplement the terms with evidence of trade usage, course of dealings, or course of performance. Here, the vendor is not offering evidence of trade usage, course of dealing, or course of performance. Therefore, the UCC would not allow the admission of this evidence if the contract were unambiguous.

A contractor purchased a furnace from a distributor of heating systems, after the distributor recommended the furnace based on the contractor's stated needs. The bill of sale included a general disclaimer of warranties in small print on the back of the document. The contractor installed the furnace in an office building. Due to a manufacturing defect, the furnace failed to heat the building as it should have. The contractor was sued by the owner of the office building. Can the contractor assert a claim against the distributor for breach of the implied warranty of merchantability? A. No, because there is no implied warranty of merchantability for sales between merchants. B. No, because the distributor disclaimed this warranty. C. Yes, because the furnace failed to heat the building. D. Yes, because the contractor relied on the distributor's judgment.

Answer choice C is correct. Goods that are not fit for the ordinary purpose for which such goods are used are not merchantable. As a merchant seller of the furnace, the distributor can be liable for breach of the warranty of merchantability. Answer choice A is incorrect because the implied warranty of merchantability applies when the seller is a merchant dealing in goods of the kind sold. It applies even where the buyer is also a merchant. Answer choice B is incorrect because an inconspicuous general disclaimer is not sufficient to disclaim the warranty of merchantability. The disclaimer need not use the term merchantability, but it must use language that makes clear there is no implied warranty and must be conspicuous if in writing. Here, the general disclaimer (which may or may not have contained sufficient language) was not conspicuous. Answer choice D is incorrect because reliance on a seller's judgment is not a requirement of the implied warranty of merchantability, and a warranty of fitness for a particular purpose may be disclaimed in general language.

A college student's mother hired a cleaning service to clean the student's apartment once a week for six months at $75 per week. For two months, the cleaning service sent the same cleaner to clean the student's apartment every week per the contract. However, in the third month, the cleaner stopped coming to clean, and the cleaning service did not send a replacement. Does the student have legal standing to enforce the contract against the cleaning service? A. No, because the student did not provide consideration to support a contract with the cleaning service. B. No, because he was a donee beneficiary. C. Yes, because the student was an intended beneficiary of the contract between his mother and the cleaning service. D. Yes, because the student is an incidental beneficiary of the contract between his mother and the cleaning service.

Answer choice C is correct. In general, an intended beneficiary is one to whom the promise of the performance will satisfy the obligation of the promisee to pay money to the beneficiary or the circumstances indicate that the promisee intends to give the beneficiary the benefit of the promised performance. An intended beneficiary has a right to enforce the contract against the promisee. Here, the mother and the cleaning service contractually intended to benefit the student. Therefore, he has the right to enforce the contract against the cleaning service. Answer choice A is incorrect because the student, as an intended third-party beneficiary, does not have to provide consideration to the cleaning service in order to enforce the contract against the cleaning service. Answer choice B is incorrect. If the promisee entered the contract for the purpose of conferring a gift on a third party, then the third-party intended beneficiary is given the right to sue the promisor. Here, the student is an intended and donee beneficiary and would therefore have legal standing to enforce the contract. Answer choice D is incorrect. An incidental beneficiary is one who benefits from a contract even though there is no contractual intent to benefit that person. An incidental beneficiary has no rights to enforce the contract. Here, the student is an intended beneficiary.

A seller owned a small, rare book store. On June 1, the seller had a conversation with a customer about a rare bound manuscript of a novel that the seller kept in her collection. The seller told the customer that she was interested in selling the manuscript, and that if he returned to the bookstore with $500 sometime within the next month, she would sell him the manuscript. The customer left the seller his e-mail address so that they could stay in touch and he also promised to return to purchase the manuscript. On June 10, a collector visited the bookstore and discussed the same manuscript with the seller. The collector offered the seller $5,000 for the manuscript, and the seller immediately accepted. She delivered the manuscript to the collector the next day. After delivering the manuscript to the collector, the seller immediately e-mailed the customer to revoke her offer. The customer received the e-mail that evening. The next day, the customer returned to the bookstore and attempted to purchase the manuscript for $500. Has the seller properly revoked her offer? A. No, because the customer accepted the seller's offer on June 1. B. No, because the seller is a merchant of books. C. Yes, because the customer received the e-mail before attempting to accept the offer. D. Yes, because the seller sold the manuscript to the collector on June 10.

Answer choice C is correct. In general, an offer can be terminated by the offeror at any time. A revocation may be made in any reasonable manner and by any reasonable means, and it is not effective until communicated. Here, the seller communicated her revocation to the customer before he attempted to accept the offer. Answer choice A is incorrect. The seller told the customer that if he returned to the bookstore with $500 sometime within the next month, she would sell him the book. While some other method of acceptance may have sufficed, the seller's directing a manner of acceptance that included full payment indicates that the customer did not properly accept the offer on June 1 by promising that he would return to purchase the manuscript. Answer choice B is incorrect because even when the offeror is a merchant, a firm offer under the UCC still requires a writing. Here, the seller only made oral assurances that the offer would remain open. Therefore, the fact that she was a merchant of books is insufficient to make her offer irrevocable. Answer choice D is incorrect because the customer did not learn about the seller's transaction with the collector on June 10. If the offeree acquires reliable information that the offeror has taken definite action inconsistent with the offer, the offer is automatically revoked. However, when the offeree is not aware of a definite action inconsistent with the offer, no constructive revocation occurs. Because the customer was not aware of the seller's transaction with the collector on June 10, the seller's offer was not constructively revoked at that time. Therefore, the offer was not revoked until the seller sent the customer the revocation e-mail on June 11.

An electrician and a landlord executed a contract under which the electrician agreed to upgrade the electricity in the landlord's ten-unit apartment building at a cost of $10,000. The electrician began work as scheduled and completed two of the ten identical units before quarreling with the landlord. The electrician refused to complete the job. The landlord hired another electrician who completed the last eight units at $1,100 each in the same time frame as was contemplated in the landlord's contract with the electrician. If the first electrician sues the landlord for restitution damages, how much will a court likely award? A. $9,200 B. $2,000 C. $1,200 D. $0

Answer choice C is correct. Restitutionary damages restore to the plaintiff (here, the electrician) whatever benefit was conferred upon the defendant (here, the landlord) prior to the breach. If the plaintiff breached the contract, his damages are generally limited to the amount of wealth conferred upon the defendant, which would take into consideration any damages suffered by the defendant. In this case, the electrician conferred roughly $2,000 of benefit on the landlord (two-tenths of the value of the original contract, assuming the contract price reflected the value conferred), and the landlord incurred $800 in extra costs as a result of the electrician's breach (an extra $100 per unit x eight units). Accordingly, the electrician is entitled to $1,200, or the amount of benefit the electrician conferred upon the landlord. Answer choice A is incorrect because the electrician was in breach and did not substantially perform his contractual obligations. Consequently, he is not entitled to recover the full contract price ($10,000) less damages incurred by the landlord because of the electrician's breach ($100 x 8, or $800). Answer choice B is incorrect. Had the electrician sought expectation damages under the contract, he may have been entitled to $2,000 if it was determined that the contract was divisible, because he completed two out of the ten units. However, this amount fails to take into account the cost of completion. Answer choice D is incorrect because the electrician would be entitled to recover some amount of damages, namely the amount of wealth conferred upon the landlord.

A large manufacturer regularly purchased a rare earth metal from various sources in order to incorporate the metal into its products. The manufacturer entered into a contract to purchase a specific quantity of the processed metal from a supplier. The contract called for the metal to be delivered in 90 days, with payment due on delivery. Coincidentally, between the contract and the agreed-upon delivery date, the manufacturer purchased the metal from some of the sources that the supplier, itself, often turned to for the metal. As a consequence of these purchases, the supplier could only obtain half of the required quantity of the metal by the delivery date. If the supplier delivers half of the required quantity of the metal on the delivery date, will it have breached the contract? A. No, because the supplier delivered the metal that it could in good faith obtain. B. No, because the manufacturer's purchases of the metal were the cause of the supplier being unable to meet its contractual obligation. C. Yes, because the supplier failed to deliver the required amount of the metal on the delivery date. D. Yes, because merchants do not have the obligation to deal in good faith with one another.

Answer choice C is correct. The seller of goods is required to comply with the perfect tender rule. Here the supplier of a rare earth metal is a seller of goods. Since the seller did not supply the full amount called for under the contract, the manufacturer could reject the amount offered by the seller. Consequently, the seller is in breach of the contract. Answer choice A is incorrect because, although there is an implied duty of good faith in every contract, the satisfaction of that duty does not protect a party who has breached a specific contract obligation. Answer choice B is incorrect because the buyer of goods is not required, absent a specific contract provision to the contrary, to refrain from purchasing from another source the same goods that it is contractually obligated to buy from a seller, unless such a purchase is undertaken for the purpose of preventing the seller from meeting its contractual obligation (i.e., in bad faith). The facts here indicate that the manufacturer's purchase from the supplier's sources was coincidental, and therefore not in bad faith. Answer choice D is incorrect because a buyer of goods is under an obligation of fair dealing with regard to the seller of the goods, regardless of whether they are both merchants.

A seller posted an advertisement in coffee shops across the city stating, "Registered dog for sale for $5,000. I will deliver him to the home of the first to pay." Before the seller even got home, he received a call from a dog breeder, who stated that she wanted to buy the dog. She provided her credit card information to the seller, who accepted the payment, placed the dog in the car, and drove to the breeder's house. When he delivered the dog to her home, she learned the dog was female. The breeder wanted to breed puppies from within her home and already had a female dog. The seller apologized profusely that the posting included a typo, as he meant to type "deliver her to the home of the first to pay." What is the breeder's strongest argument in having the money refunded to her? A. That no contract was formed because there was only an advertisement, not an offer. B. The seller committed fraud. C. The seller caused the buyer's confusion. D. No performance occurred until the breeder accepted the dog.

Answer choice C is correct. When one party is mistaken as to an essential element of the contract, the mistaken party can void the contract if she did not bear the risk of mistake and the non-mistaken party caused the mistake, had a duty to disclose or failed to disclose the mistake, or knew or should have known that the other party was mistaken. Here, the seller caused the mistake by using the wrong pronoun to describe the dog, failed to disclose the mistake until after payment was accepted, and should have known that the buyer would be mistaken based on the advertisement. Moreover, nothing indicates that the buyer bore any risk of mistake. The dog's gender was certainly an essential term for the buyer, as she wanted the dog purely for breeding purposes. Answer choice A is incorrect because the seller's communication was, in fact, an offer, not simply an advertisement. Advertisements are generally considered to be invitations to receive offers from the public, rather than offers. However, an advertisement that is specific and limiting can constitute an offer. Here, the buyer's post was specific—it identified a specific item for sale for a specific price; it was also limited—only the first person to respond was eligible. Accordingly, a contract was made between the buyer and seller. Note that, as this contract is for a sale of a good that is over $500, which would normally fall within the Statute of Frauds, there is an exception to the writing requirement when payment has been made and accepted, as here. Answer choice B is incorrect because, although the seller did make a misrepresentation, there is insufficient evidence to prove that he was fraudulent. Accordingly, the buyer would have the most success voiding the contract on the basis of mistake. Because the elements of mistake are more easily proved than fraud, this is not the best answer. Answer choice D is incorrect because both parties did perform.

A dog walker and a dog owner entered into negotiations for the walker to provide twice daily walks for the owner's dachshund. After several emails back and forth, the owner and the walker agreed in writing that the walker would walk the dachshund at 10:00 a.m. and 3:00 p.m., Monday through Friday, for $15 per walk. The walker walked the dachshund as specified for a week. When the owner returned home from work in the evenings, however, she found that the dachshund's food and water bowls were always empty. She emailed the walker, explained that the dachshund had diabetes, and asked the walker to fill the bowls after each walk. The walker replied to the email and agreed to fill the bowls after each walk. Two weeks later, the owner returned home from work to find the dachshund in a diabetic coma; the dog's blood sugar had dropped because she had run a lot on her walk and then did not have any food in her bowl after the walk. She sued the walker for her veterinary expenses, citing the email in which the walker had agreed to fill the bowls after each walk.Will the court likely admit evidence of the walker's agreement to fill the bowls after each walk? A.No, because the contract between the parties did not include a requirement to fill the bowls. B. No, because the agreement to fill the bowls arose after the contract was executed. C. Yes, because the parol evidence rule does not apply. D. Yes, because there was no new consideration for the bowl filling.

Answer choice C is correct. While the parol evidence rule generally prevents a party to a written contract from presenting prior or contemporaneous extrinsic evidence that contradicts or is inconsistent with the terms of the contract as written, it does not apply to subsequent agreements like the email agreement to fill the bowls. Answer choice B is therefore incorrect. Answer choice A is incorrect because, even though the original contract did not include a requirement to fill the bowls, the parties subsequently agreed that the walker would do so. Answer choice D is incorrect because, while it is true that the court might not enforce the bowl-filling requirement because the owner did not pay additional consideration, this question asks about whether the agreement to fill the bowls would be admitted, not whether it would be enforced.

A farmer contracted with a fruit distributor to sell the distributor 1,000 bushels of peaches at $10 per bushel, the going market price for peaches. After the parties executed the contract, a late frost in a neighboring state completely destroyed that state's peach crop. The affected state normally produced over half the peaches grown in the country. In response, the market price of peaches spiked to over $50 per bushel. The farmer attempted to renegotiate with the distributor, who refused to modify the contract. Thereafter, the farmer refused to deliver the peaches as scheduled. The distributor sued the farmer, who raised the defense that to force him to sell his produce so far below market value would be unconscionable. Does the farmer have a valid defense? A. Yes, because no reasonable person would agree to sell peaches at one-fifth of market price. B. Yes, because the frost was unforeseeable. C. No, because the farmer and the distributor had fairly equal bargaining power. D. No, because the contract price was reasonable when the farmer agreed to it.

Answer choice D is correct. A court may modify or refuse to enforce a contract or part of a contract on the ground that it is unconscionable, that is, when it is so unfair to one party that no reasonable person in the position of the parties would have agreed to it. The contract or part of the contract at issue must have been offensive at the time it was made. Here, the farmer contracted with the distributor to sell peaches at $10 per bushel, which was a fair market price at the time the parties entered the contract. The fact that intervening events caused prices to rise only means that the farmer did not receive as good a deal as he could have if he had waited; this, however, is the risk inherent in business dealings and does not make the transaction unconscionable. Answer choice A is incorrect because this addresses only half of the analysis; the unfairness must also exist at the time of contracting. Answer choice B is incorrect because foreseeability is not a relevant factor in an unconscionability inquiry. Answer choice C is incorrect because, while the defense of unconscionability is often used when there is a contract between parties with vastly unequal bargaining power, that is not the only way that a contract could be unconscionable. In other words, this contract still could have been unconscionable, even if the bargaining power of the two parties was relatively equal.

A manufacturer of portable X-ray machines contracted with a healthcare system to provide 25 custom-made portable X-ray machines to the system's member hospitals, at a total cost of $3,000,000, to be delivered within sixty days. The machines use an isotope of the rare earth metal thulium as their radiation source. Without thulium, the portable X-ray machines cannot be manufactured. Several days after the manufacturer and healthcare system entered into the agreement, unprecedented rains in southern China caused a massive mudslide that destroyed operations at the only commercial thulium mine in the world. The mine owner stated that operations would not be restored for at least ninety days. The manufacturer subsequently informed the healthcare system that because of the sudden market shortage of thulium, it would not be able to deliver the portable X-ray machines in the time frame agreed upon in the contract. Sixty-one days after the contract was signed, the healthcare system filed a complaint alleging breach of contract. Will the healthcare system likely prevail in its suit against the manufacturer? A. Yes, because the manufacturer assumed the risk of the occurrence of a thulium shortage. B. Yes, because the UCC perfect tender rule requires strict performance of the terms of the contract. C. No, because the parties had made a mutual mistake. D. No, because performance was impracticable.

Answer choice D is correct. A party's duty to perform can be discharged by impracticability. The defense of impracticability is available if: (i) an unforeseen event occurs; (ii) the nonoccurrence of which was a basic assumption on which the contract was based; and (iii) the party seeking discharge is not at fault. Here, the weather conditions in southern China were unprecedented, and the parties could not have foreseen a massive mudslide that destroyed the only thulium supply, making it impossible for the manufacturer to produce more portable X-ray machines. The manufacturer was not at fault for the weather in China, and therefore, would be able to raise a valid defense of impracticability. Answer choice A is incorrect because the facts do not indicate that the manufacturer had assumed the risk of a world shortage of thulium, either expressly or impliedly. Answer choice B is incorrect because although the UCC has adopted the perfect tender rule, it also recognizes the defense of impracticability, which discharges the manufacturer's duties in this case. Answer choice C is incorrect because in order for mutual mistake to apply, the mistaken fact must have been in existence at the time the contract was formed.

An undergraduate student planned to transfer to a university in the town where her aunt resided and to live in an on-campus dorm. She told her aunt about her plans in an email. The next day, the aunt replied with an email containing the following: "I am so excited! If you don't want to deal with a roommate, don't apply for a room in the dorms, you can just stay in my spare room rent-free!" Elated, the student immediately submitted her transfer application without submitting an application for a dorm room on campus. After the deadline to apply for a dorm room had passed, the student's aunt called and explained that she had no room for the student because she was renting out her spare room to another student who was willing to pay rent. The student had to sign a lease in an off-campus apartment and ended up paying much more than she would have paid to live in the dorm. If the student sues her aunt to recover damages, is the student likely to recover? A. No, because the aunt's promise to provide a rent-free room is an unenforceable gift promise. B. No, because the student did not effectively accept the aunt's offer before it was revoked. C. Yes, because the agreement was in writing and the apartment was more expensive than the dorm. D. Yes, because the student decided not to apply for a dorm room in reasonable reliance on her aunt's promise to provide her a room.

Answer choice D is correct. A promise is binding if the promisor should reasonably expect it to induce action or forbearance on the part of the promisee or a third person, the promise does induce such action or forbearance, and injustice can be avoided only by enforcement of the promise. Here, the student reasonably relied on her aunt's promise of rent-free boarding when she did not apply for on-campus housing. Therefore, the student will likely succeed in her action against her aunt. Answer choice A is incorrect because the student has reasonably relied on this promise to her detriment. Therefore, the promise is enforceable under the doctrine of promissory estoppel. Answer choice B is incorrect because the student did not expressly need to accept the aunt's promise to provide her with housing. The aunt specifically told her not to apply for on-campus housing if she did not want to have a roommate, and promised that she would have a rent-free room if she wanted it. Because the student reasonably relied on this promise to her detriment, the promise is enforceable. Answer choice C is incorrect because these facts are insufficient to create a consideration substitute for an otherwise unenforceable promise.

A mining company contracted with a railroad to transport 10,000 tons of coal from the company's mines to a power company at a cost of $100,000. The railroad told the mining company that the coal would arrive at the power company on June 1, but the contract contained a clause that the railroad would not be liable for any losses suffered by the mining company as a result of a late shipment. The railroad was aware that the mining company had contracted with the power company to deliver the coal on June 1, and pursuant to standard industry custom, the price to be paid by the power company decreased by $1 per ton for each day that the coal was late. The shipment of coal did not reach the power company until June 11, and the railroad had no justification for the 10-day delay. Because of the delay, the mining company lost $100,000 in revenue from the sale. The mining company filed suit against the railroad for breach of contract, claiming $100,000 in damages. Is the mining company likely to succeed in its claim? A. company would suffer from the railroad's delay were known to the railroad prior to shipment of the coal. B. Yes, because consequential damages cannot be excluded by a merchant. C. No, because the claimed damages are disproportionate to the original contract price between the railroad and the mining company. D. No, because the contract between the mining company and the railroad protected the railroad from losses suffered by the mining company due to a late shipment.

Answer choice D is correct. Although a party may be liable for consequential damages of the other party to a contract where those damages are foreseeable, a party may eliminate that liability through an agreement with the other party. Here, while the railroad was aware of the mining company's liability for failing to supply the power company with coal on June 1st, the contract between the railroad and the mining company eliminated the railroad's liability for any losses suffered by the mining company due to late delivery. Answer choice A is incorrect because the parties to a contract are generally free to allocate responsibility for consequential damages as they see fit. So even though the mining company's liability due to a late shipment was known to the railroad, the railroad expressly disavowed responsibility for that liability. Answer choice B is incorrect because, although the UCC does prohibit the limitation or exclusion of liability for consequential damages, this prohibition applies only where limitation or exclusion is unconscionable, and a commercial loss is not prima facia unconscionable. In addition, the contract at issue (i.e., between the railroad and the mining company) is not a sale of goods and hence not governed by the UCC. Answer choice C is incorrect because, while the lack of proportionality between the contract price and the amount of consequential damages may be an underlying factor in determining whether such damages are foreseeable, proportionality alone is not determinative as to whether such damages are foreseeable. In any case, the parties may generally allocate responsibility for consequential damages as they see fit.

A paving company entered into a contract with a real estate developer to repave a large parking lot in the developer's shopping center. Since the paving company wanted to establish a good reputation in the market, it discounted its price. The paving company expected to make $20,000 in profit on the contract.Midway through the project, the developer notified the paving company to cease work; the paving company immediately complied. At the time of the notice, the paving company had incurred costs of $200,000. The cost of hiring another contractor to perform the same work would have been $225,000. The paving work performed had increased the value of the shopping center by $100,000. The paving company filed a lawsuit against the developer. The fact finder determined that the developer's repudiation of the contract was without justification. What is the maximum amount of damages the paving company can be awarded? A. $100,000, the amount by which the value of the shopping center was increased. B. $200,000, the costs incurred by the paving company. C. $220,000, the paving company's expectancy damages. D. $225,000, the amount that the developer would have had to pay for the portion of the paving job that was completed.

Answer choice D is correct. Although expectancy damages normally are awarded in a breach-of-contract action, restitutionary damages are permitted in cases where the nonbreaching party has partially performed a below-market-price contract. Otherwise, the breaching party would profit from its breach. Consequently, the paving company may recover the benefit conferred upon the developer as measured by the amount the developer would have had to pay to secure the same performance as that rendered by the paving company. Answer choice A is incorrect because, although the increase in the defendant's property value due to the plaintiff's performance is one measure of restitutionary damages, that amount is less than the paving company's restitutionary damages as measured by the cost to the developer of obtaining the same performance rendered by the paving company. Answer choice B is incorrect because the paving company's reliance damages are exceeded by both its expectancy and restitutionary damages. Answer choice C is incorrect because the paving company's expectancy damages, based on the cost incurred by the company plus its profit on the contract, are less than the company's restitutionary damages.

A new homeowner was shopping with a friend at an appliance store for a new refrigerator. The friend, having just purchased a new refrigerator herself, had done extensive research about the various brands and models of refrigerators. She recommended a particular brand and model based on her knowledge of the homeowner's needs. Although the store did not have that model in stock, the friend showed the homeowner a different model from the same manufacturer, pointing out that the recommended refrigerator was exactly like that model. Relying on her friend's advice, the homeowner ordered the recommended refrigerator. When the refrigerator was delivered, the homeowner discovered that the model she had ordered was considerably different from the model that her friend had shown her at the store and did not have several features the homeowner desired. Under which of the following warranties can the homeowner sue to recover from her friend? A. Express warranty only. B. Implied warranty of merchantability only. C. Both express warranty and implied warranty of merchantability. D. Neither express warranty nor implied warranty of merchantability.

Answer choice D is correct. Although the friend's conduct would create both an express warranty and an implied warranty of merchantability if the friend were the seller of the refrigerator, since the friend is not, neither warranty applies. Answer choices A, B, and C are incorrect because the friend is not a seller of the refrigerator and therefore her comments do not constitute an express warranty or an implied warranty of merchantability.

A grocery store and a greenhouse farmer entered into a valid contract for100 boxes of Grade 1 tomatoes, to be delivered on or before the end of the month. On the last day of the month, the farmer delivered 100 boxes of Grade 2 tomatoes to the store. Based on the price and popularity of Grade 2 tomatoes in the area, the farmer reasonably believed that the store would accept these tomatoes at a reduced price. When the grocery store refused to accept the delivery, the farmer promised to return the following morning with 100 boxes of Grade 1 tomatoes. When the farmer did, the grocery store again refused to accept the tomatoes.Is the store's rejection of the Grade 1 tomatoes proper? A. Yes, because their delivery was untimely. B. Yes, because the perfect tender rule applies to sales of goods. C. No, because the store's earlier rejection of the Grade 2 tomatoes at a reduced price was improper. D. No, because the farmer cured the nonconforming tender.

Answer choice D is correct. Although the perfect tender rule generally applies to a sale of goods, the seller has a right to cure a defective tender—even after the time for performance under the contract has elapsed—if the seller had reasonable grounds to believe that the buyer would accept the goods despite the nonconformity. In curing the defect, the seller must notify the buyer of its intent to do so and effect the cure within a reasonable time. Although the farmer delivered the Grade 1 tomatoes to the store after the time for performance had passed and thus failed to satisfy the perfect tender rule, because the farmer had reasonable grounds to believe that the store would accept the Grade 2 tomatoes at a reduced price, the farmer had a reasonable time to correct the defective tender. Because the farmer delivered the conforming tomatoes the following day, it is likely that the store's rejection of the tomatoes was improper. Answer choice A is incorrect because, although the farmer's delivery of the Grade 1 tomatoes was after the time specified in the contract, the farmer's reasonable belief that the store would accept the Grade 2 tomatoes allowed the farmer a reasonable time in which to cure the initial defective tender. Answer choice B is incorrect because, although the perfect tender rule does apply to sales of goods that are not sold under an installment contract and the farmer did not timely make a perfect tender, the farmer had a reasonable time in which to cure the initial defective tender. Answer choice C is incorrect because the store's earlier rejection of the Grade 2 tomatoes was not improper since the farmer failed to tender Grade 1 tomatoes. Although the farmer may have had reasonable grounds for believing that the store would accept the Grade 2 tomatoes at a reduced price, this belief alone does not compel the store to accept those tomatoes.

A restaurant owner sent a signed order form to a produce supplier that read: "Please ship us 100 pounds of potatoes at your current price." The supplier received the order form on November 9. Later the same day, the supplier mailed the owner a letter with the proper address and postage. The letter stated that the order had been accepted at the supplier's current price for potatoes. On November 10, before receiving the supplier's reply, the owner telephoned the supplier to inform the supplier that the owner had found a closer supplier and was canceling the order. The supplier asked the owner to reconsider, but the owner refused. On November 11, the owner received the supplier's letter. Relying on the owner's telephone call, the supplier never shipped any potatoes to the restaurant. As of November 12, is there an enforceable contract between the supplier and the owner? A. No, because the owner effectively revoked its offer before it was properly accepted. B. No, because the owner's offer could be accepted only by shipment of the goods, and the supplier never shipped the potatoes. C. Yes, because the owner's offer was irrevocable for a reasonable time. D. Yes, because the supplier effectively accepted the owner's offer before the owner revoked it.

Answer choice D is correct. An acceptance that is mailed within the allotted response time is effective when sent rather than upon receipt, unless the offer provides otherwise. The mailing must be properly addressed and include correct postage. Here, those requirements are met. Offers revoked by the offeror are effective upon receipt. Therefore, the supplier effectively accepted the offer before receiving the owner's revocation of the offer, and a contract was formed upon acceptance. Answer choice A is incorrect because the supplier properly mailed its acceptance of the offer before the owner revoked the offer. Therefore, the owner's power to terminate the offer had ended, and an enforceable contract exists. Answer choice B is incorrect. If a buyer requests that the goods be shipped, then the buyer's request will be construed as inviting acceptance by the seller either by a promise to ship or by prompt shipment of conforming or nonconforming goods. Therefore, a promise to ship the potatoes here serves as an effective acceptance of the offer. Answer choice C is incorrect because there was no signed writing assuring that the offer would remain open for any period of time. Under the UCC, the offer was therefore revocable.

A daughter successfully petitioned a court to have her father declared incompetent to manage his affairs and to have herself appointed as guardian of his property. Subsequently, the father ordered furniture totaling $3,500 from a local store. The store, unaware of the guardianship and not otherwise having a reason from the father's behavior to learn of his incompetency, delivered the furniture to the father's residence where he received and accepted it. The next day a flood destroyed the furniture before the daughter had the opportunity to contact the store. Is the store entitled to enforce the contract for the sale of the furniture? A. Yes, because the risk of loss had passed to the father, as buyer of the furniture, upon its delivery. B. Yes, because the store was unaware of the guardianship and the father's incompetency. C. No, because the daughter did not have the opportunity to contact the store. D. No, because the father had been adjudicated incompetent.

Answer choice D is correct. An individual who is the subject of a court-ordered guardianship over that individual's property lacks the capacity to enter into a contract. Consequently, any contract purportedly entered into by such an individual is void. Since the father was under a court-ordered guardianship, his contract to purchase the furniture was void. (Note: If the furniture is a necessity, the store may be able to recover the furniture's reasonable value in restitution under a quasi-contract action, but cannot enforce the contract of sale between the store and the father.) Answer choice A is incorrect because, although where a contract for the sale of goods exists the delivery of the goods by the seller to the buyer generally shifts the risk of loss to the buyer, here the purported contract entered into by the father for the purchase of the furniture was void. Answer choice B is incorrect because, where a guardianship over an individual's property has been recognized by a court, the seller's lack of actual knowledge of the guardianship or the individual's inability to contract is irrelevant. Answer choice C is incorrect because, since the father's contract with the store was void rather than merely voidable, the daughter's lack of opportunity to contact the store is irrelevant.

A lightbulb manufacturer was trying to develop a new way to cool the glass for its industrial lights to make the glass lighter and stronger. On March 1, the manufacturer contracted in writing with a home improvement store to sell and deliver to the store 100 units of industrial light meeting the store's specifications and using the new glass. The delivery clause provided that the manufacturer would deliver the lights on or before May 15. As of March 1, the manufacturer had not yet successfully produced any lightbulbs using its new method, but it was confident that is was close to a breakthrough. However, on April 10, the manufacturer's development team concluded that the method they were developing consistently created glass unsuitable for industrial lightbulbs. The manufacturer immediately notified the store that, due to the present technological limits in the art of glass manufacturing, it was impossible for the manufacturer or any other company to fulfill an order of lightbulbs meeting the store's requested specifications. If the store sues the manufacturer for breach of the March 1 contract, which party is likely to prevail? A. The manufacturer, because it was objectively impossible to perform the manufacturer's duty under the contract. B. The manufacturer, because the manufacturer acted in good faith and used its best efforts to perfect the method under development. C. The store, because the defense of impossibility does not apply to merchants under the UCC. D. The store, because the manufacturer assumed the risk that the new process would not produce suitable glass for lightbulbs.

Answer choice D is correct. If a party assumes the risk of an event happening that makes performance impracticable, then the defense of impracticability will not apply. Here, the manufacturer knew at the time of contracting that the process had not yet been perfected. Based on these facts, the manufacturer assumed the risk that it might never successfully perfect the new method, and that the order would be impossible to fill. Therefore, the defense of impracticability will not apply. Answer choice A is incorrect because mere objective impossibility is not enough to provide a defense against a breach of contract action. For the defense of impracticability to be available, an unforeseeable event must occur, nonoccurrence of the event must be a basic assumption of the contract, and the party seeking discharge of a breached duty must not be at fault. Here, the failure of the manufacturer to perfect the new glass-cooling method was foreseeable, and the manufacturer therefore assumed the risk that it may fail to develop the promised glass. Answer choice B is incorrect because acting with good faith and using one's best efforts to avoid a breach are not defenses to a breach of contract claim. Answer choice C is incorrect. First, note that under the modern view, the defense traditionally called "impossibility" is now generally referred to as "impracticability." Second, this defense does apply to merchants under the UCC.

On behalf of an elementary school, the school's principal entered into a written contract to purchase shirts for the school's students for a total cost of $5,000. The name of the school was to be imprinted on the back of each shirt. After the seller had acquired the shirts but before they had been imprinted, the principal emailed the seller and requested, in good faith, that a picture of the school's mascot be imprinted on the front of the shirts at no additional cost. In a reply email, the seller agreed to the principal's request. When the shirts arrived at the school, only the school's name appeared on the back of each shirt; the school's mascot did not appear on the shirt. The principal rejected the shirts and refused to pay for them. The seller sued the school for breach of contract. Who will prevail? A. The seller, because the shirts were specially manufactured goods. B. The seller, because the seller did not receive consideration for the modification. C. The school, because the school was not a merchant. D. The school, because of the perfect tender rule.

Answer choice D is correct. In a sale-of-goods transaction, such as the purchase of shirts, the Uniform Commercial Code (UCC) requires that the goods tendered by the seller conform in all respects to the contract. Although the shirts shipped to the school conformed to the specifications of the original contract, this contract was modified by the subsequent emails between the principal and the seller. Answer choice A is incorrect because the fact that the shirts were specially manufactured goods is relevant only when the buyer is claiming the statute of frauds as a defense. The fact that the goods were specially manufactured is irrelevant to the validity of the written agreement to modify the original contract in this case. Answer choice B is incorrect because the UCC, rejecting the preexisting duty rule, does not require consideration in order for the good-faith modification of a contract to be valid. Answer choice C is incorrect because the school's status as a non-merchant is irrelevant to determining the terms of this contract or its breach.

On March 1, an office supply vendor emailed a manufacturer asking the following: "We have received your catalog and are interested in doing business. Do you offer any discounts for bulk orders of printer ink?" On March 2, the manufacturer emailed the vendor the following reply: "For a limited time, we can offer you 1000 units of printer ink for 25% off the unit price." The vendor replied with the following by email on March 4: "We accept. Please deliver the printer ink to our address on March 30." The manufacturer replied with the following on the same day: "As noted in the Terms of Service in our catalog, we do not offer shipping. By March 20, you will need to choose one of the remote offices listed in our catalog. You can pick up the ink there on March 30." On March 5, the vendor emailed the following to the manufacturer: "At your prices, it is outrageous to refuse to offer shipping. We won't be doing business." The manufacturer never replied. On March 15, the vendor sent the following email to the manufacturer: "You drive a hard bargain. We will pick up the shipment at the closest remote office on March 30." If the manufacturer does not have the ink available by March 30, can the vendor recover in an action for breach of contract? A. No, because the manufacturer's price quote on March 2 was not an offer to form a contract. B. No, because the vendor clearly and unequivocally repudiated the contract on March 5. C. Yes, because both of the parties are merchants. D. Yes, because the vendor properly retracted its repudiation on March 15.

Answer choice D is correct. Repudiation may be retracted so long as the other party has not canceled the contract or materially changed position. Here, the manufacturer did not reply to the March 5 email and does not appear to have changed position based on the repudiation in any way. Therefore, the retraction is effective, and the vendor can enforce the contract. Answer choice A is incorrect because the price quote in the March 2 email was more than an advertisement or an invitation to deal. It was an objective manifestation of a willingness by the offeror to enter into an agreement, creating the power of acceptance in the offeree. Answer choice B is incorrect because the vendor effectively retracted its repudiation with its March 15 email. Answer choice C is incorrect. Even if the vendor's request for shipping was a new term under a "battle of the forms" analysis, the manufacturer promptly and clearly objected to the term, and the vendor then repudiated on the contract. Therefore, the fact that the parties are merchants is not determinative of the vendor's ability to bring an action to enforce this contract.

Completing an online order form, a customer ordered a handmade colored glass ornament to hang in the window of her home. In a box labeled "Comments," the customer wrote, "red, please." Via email, the online retailer sent a notice acknowledging the order, but reserving the right to send an ornament in any color. The retailer shipped the customer a green ornament. Which of the following arguments would not support the customer's position that she does not have to pay for the ornament? A. The retailer shipped nonconforming goods. B. The customer did not separately agree to receive an ornament in any color. C. Because customer was not a merchant, the retailer's additional term in acknowledgment is not part of contract. D. The retailer's acceptance did not mirror the customer's offer.

Answer choice D is correct. The mirror image rule, which states that the acceptance must mirror the terms of the offer, does not apply to a sale of goods, such as a glass ornament. Answer choices A, B, and C are incorrect because they all support the customer's position that she does not have to pay for the ornament. Answer choice A is incorrect because the failure of the retailer to ship conforming goods (i.e., a red ornament) would constitute a breach of the retailer's contract obligations. Answer choice B is incorrect. Because the customer is not a merchant, any additional terms in the acceptance are treated as a proposal. This proposal must be separately accepted by the customer to become part of the contract. Answer choice C is incorrect because, if the acknowledgment is an acceptance, since the customer is not a merchant, a different term (i.e., an ornament of any color) is not part of the contract.

A retail store that specialized in crystal figurines entered into a contract to purchase 100 crystal swans at a price of $50 each from the artisan who would design and make them. The store paid for the swans at the time that the contract was executed. The contract specified that the artisan would deliver the swans personally to the store. The artisan notified the store upon completion of the swans in conformity with the contract. Unfortunately, 10 of the swans were destroyed in transit. The store refused to accept the remaining 90 swans, even though the artisan offered to reduce the purchase price by $500. The store sued the artisan for the return of the $5,000 paid to the artisan. What will be the likely result? A. The artisan will recover the entire contract price, because risk of loss transferred to the store upon the artisan's notification to the store of the completion of the crystal swans. B. The artisan will prevail, but can only recover the contract price for the intact crystal swans. C. The store will prevail, because the contract is void due to the artisan's inability to deliver 100 swans. D. You Selected: The store will prevail, because the store is not required to accept a non-conforming delivery.

Answer choice D is correct. The store has the option of voiding the contract because the artisan failed to deliver all 100 of the crystal swans as called for in the contract. Answer choice A is incorrect because, since the artisan was a merchant and the contract did not require or authorize the swans to be shipped by carrier, the risk of loss did not pass to the store until the store received the swans. Answer choice B is incorrect because the artisan cannot force the store to accept a non-conforming shipment. Answer choice C is incorrect because the contract is not void. The buyer may accept delivery of the swans at the lower price or avoid the bargain. Consequently, the contract is voidable, but not void.

A borrower failed to make several payments due on a business loan. While the borrower was in the hospital recovering from a major operation, the lender, who was a family friend, approached the borrower's wife about additional security for the business loan. The lender intimated that, if some action was not taken with regard to the loan, the lender would have to file a civil action. Distraught over her husband's physical condition, the borrower's wife, at the lender's suggestion, granted the lender a mortgage on valuable property that she had recently inherited. The mortgaged property was to serve as additional security for the business loan. As a consequence of the mortgage, the lender did not reduce the outstanding amount of the business loan but did extend the time in which the borrower had to repay the loan. Of the following, which would be the best basis on which the borrower's wife can seek to avoid the mortgage? A. The borrower's wife granted the mortgage under duress. B. The borrower's wife lacked the capacity to enter into a contract. C. The lender did not give additional consideration for the mortgage. D. The mortgage was obtained through undue influence.

Answer choice D is correct. Undue influence is the unfair persuasion of a party to assent to a contract. Here, the circumstances surrounding the mortgage, such as the mortgagor's emotional trauma over her husband's physical condition, the friendship between the mortgagee and the mortgagor, the granting of the mortgage at the lender's suggestion, and the failure of the mortgagee to reduce the outstanding amount of the business loan despite the mortgaged property being valuable, all point to the mortgage being obtained through undue influence. While undue influence is not a certainty, it is the best available avenue for the wife to pursue rescission. Answer choice A is incorrect because, in order for a contract to be voidable for duress, there must be an improper threat that deprives a party of a meaningful choice. Generally, the threat of civil action is not improper unless it is made in bad faith. Here, the lender did not act in bad faith to compel the wife to grant the mortgage in a transaction unrelated to the loan, but instead sought the mortgage as additional security for the loan. Answer choice B is incorrect because emotional distress does not generally rise to the level of incapacity to contract. In order to challenge a contract on the grounds of mental incapacity, the party must have been unable to understand the nature and consequences of the transaction or unable to act in a reasonable manner with regard to the transaction and the other party had reason to know of this fact. Answer choice C is incorrect. Although the lender (mortgagee) had already made the loan to the borrower and the lender did not provide consideration for the mortgage by reducing the outstanding amount of the loan, the lender did extend the time for the payment of that loan. As a consequence, the lender did give additional consideration for mortgagor's granting the mortgage. Although this consideration directly benefited the mortgagor's husband rather than the mortgagor, a party's direct benefit from a contract need not be financial.

A lumberyard contracted with a retail home improvement company to provide the company with pine boards of various lengths and amounts. The contract was silent as to delegation, and the lumberyard, acting in good faith, delegated its duties under the contract to a third party. After asking for, and receiving, assurances from the third party that the boards would conform to contract specifications, the company accepted the delegation. The third party delivered the boards, which the company rejected because the shipment did not conform to its specifications. The company sued both the lumberyard and the third party. The lumberyard moved to dismiss the claims against it.Should the court dismiss the suit against the lumberyard? A. Yes, because the company accepted the delegation. B. Yes, because the lumberyard delegated its contractual duties in good faith. C. No, because this sale of goods contract did not specifically allow for the delegation of duties. D. No, because a delegation of duties does not absolve the lumberyard of its contractual liability.

Answer choice D is correct. When obligations are delegated, the delegator is not released from liability, and recovery can be had against the delegator if the delegate does not perform, unless the other party to the contract agrees to release that party and substitute a new one (a novation). Consequently, the lumberyard remained liable on the contract. Answer choice A is incorrect because, while a party to a contract may be released from liability if the other party agrees to release that party and substitute a new one (a novation), mere acceptance of a delegation does not constitute a novation, and the lumberyard remained liable on the contract. Answer choice B is incorrect because the mere fact that a party to a contract delegates its duties in good faith does not eliminate its duties to perform under the contract. Answer choice C is incorrect because it misstates the law. There is no UCC provision that prevents the delegation of duties unless the contract specifically provides for it.

A 90-year-old woman and a 21-year-old nurse entered into an agreement that the nurse would feed, bathe, and otherwise care for the elderly woman for $20,000 per week plus room and board. They set their agreement down in writing. When the woman's children learned of the contract, they refused to pay. The nurse sued, arguing that the writing was enforceable because it was totally integrated and in writing. The children argued that, based on prior conversations between their mother and the nurse in which the mother said the year was 1956 and the President was Eisenhower, the nurse knew that their mother was suffering from dementia. Will the court likely admit evidence of the prior conversations? A. No, because the writing was a totally integrated agreement. B. No, because the elderly woman signed the agreement. C. Yes, because the parol evidence rule applies. D. Yes, because the evidence is being offered as a defense to formation of the contract.

Answer choice D is correct. While the parol evidence rule generally prevents a party to a written contract from presenting prior or contemporaneous extrinsic evidence that contradicts or is inconsistent with the terms of the contract as written, the rule does not apply to evidence offered to establish a defense such as incompetence. Because if the elderly woman was suffering from dementia, she would have been incompetent to enter into the contract, the court will likely admit evidence of the prior conversations. Answer choice C is therefore incorrect. Answer choices A and B are incorrect because, even if the agreement was totally integrated and the elderly woman signed the writing, her incompetence would invalidate the agreement.


संबंधित स्टडी सेट्स

Neat pathognomonics and cool stuff

View Set

Business Innovation Final Set #2

View Set

Chapter 01 - A Preview of Cell Biology

View Set

Physical Science (PSY151) Chapter 5: Les Thomas

View Set

Econ Unit 6 Fiscal Policy: Government Taxation and Spending

View Set

chapter 11 sensation and perception

View Set